Download as pdf or txt
Download as pdf or txt
You are on page 1of 88

Copyright 2014 Delhi Academy of Medical Sciences, All Rights Reserved.

1/88
Copyright 2014 Delhi Academy of Medical Sciences, All Rights Reserved. 2/88
Test Information
Test Name Pharmacology including Pharmacogenetics Total Questions 200
Test Type Examination Difficulty Level Difficult
Total Marks 600 Duration 120minutes

Test Question Language:- ENGLISH

(1). The receptors for noradrenaline, adrenaline, dopamine all belong to which category -
a. Seven-pass receptors
b. Ligand-gated receptors
c. Four-pass receptors
d. Single-pass receptors

Solution. Ans-1: (a) Seven-pass receptors
Ref:Read the text below
Sol:
Catecholamines(adrenaline, noradrenaline and dopamine) act through G protein coupled receptors which belong to a large
superfamily of seven pass transmembrane protein (also k/a serpentine receptors).
These proteins span the cell membrane seve times.
Answer. a

Copyright 2014 Delhi Academy of Medical Sciences, All Rights Reserved. 3/88
(2). In erectile dysfunction, which of the following drug is not used?
a. Apomorphine
b. Phenylephrine
c. Sildenafil
d. Papaverine

Solution.
Ans-2: (b) Phenylephrine
Ref:Read the text below
Sol:
List of drugs causing erectile dysfunction

Classification Drugs
Diuretics
Thiazides
Spironolactone
Antihypertensives
Calcium channel blockers
Methyldopa
Clonidine
Reserpine
-Blockers
Guanethidine
Cardiac/anti-hyperlipidemics
Digoxin
Gemfibrozil
Clofibrate
Antidepressants
Selective serotonin reuptake inhibitors
Tricyclic antidepressants
Lithium
Monoamine oxidase inhibitors
Tranquilizers
Butyrophenones
Phenothiazines
H
2
antagonists

Ranitidine
Cimetidine
Hormones
Progesterone
Estrogens
Corticosteroids
GnRH agonists
5-Reductase inhibitors
Cyproterone acetate
Cytotoxic agents
Cyclophosphamide
Methotrexate
Roferon-A
Anticholinergics
Disopyramide
Anticonvulsants
Recreational
Ethanol
Cocaine
Marijuana

Answer. b

Copyright 2014 Delhi Academy of Medical Sciences, All Rights Reserved. 4/88
(3). A73-year-old patient has heart failure that has worsened over the past 12 months. Which of the following drugs is likely to slow
progression even though it has no direct positive or negative effects on cardiac contractility?
a. Digoxin
b. Dobutamine
c. Losartan
d. Nesiritide

Solution. Ans-3: (c) Losartan
Ref:Read the text below
Sol:
Losartan, an AT1 angiotensin receptor antagonist, has been shown to slow the progression of heart failure, in common with the
ACE inhibitors, beta blockers, and spironolactone, which also slow progression of this disease.
Positive inotropic agentssuch as digoxin and dobutamine do not slow progression although they may be useful for reducing
symptoms.
Nesiritide is a peptide that causes vasodilation and sodium diuresis and may have value in acute but not chronic failure.
Answer. c

(4). Which of the following drugs is associated with discoloration of the skin, photosensitivity, thyroid dysfunction, and pulmonary
complications?
a. Amiodarone
b. Diltiazem
c. Ibutilide
d. Lidocaine

Solution. Ans-4: (a) Amiodarone
Ref:Read the text below
Sol:
Amiodarone is a large, insoluble molecule that deposits in skin(and the cornea) and causes photosensitivity.
It contains two iodine atoms, which may be responsible for binding to thyroid receptors and the observed thyroid problems.
It is associated with pulmonary fibrosis (sometimes fatal), but the mechanism is unknown.
In spite of these adverse effects, it is extremely useful in arrhythmia treatment.
Diltiazem is associatedwith constipation and cardiac depression; ibutilide with torsade de pointe arrhythmia,lidocaine with
convulsions, and procainamide with torsade de pointe arrhythmia and drug-induced lupus.
Answer. a

(5). Which of the following drugs causes hyperchloremic metabolic acidosis and may cause hyperammonemia in a patient with cirrhosis?
a. Acetazolamide
b. Amiloride
c. Furosemide
d. Hydrochlorothiazide

Solution. Ans-5: (a) Acetazolamide
Ref:Read the text below
Sol:
Acetazolamide acts in the proximal tubule to block carbonic anhydrase, greatly increasing bicarbonate excretion and alkalinizing
the urine while causing metabolic acidosis.
In cirrhosis, ammonia is excreted in larger amounts because urea production in the liver is limited.
If an acid urine (necessary for converting ammonia to ammonium ion) cannot be produced, the ammonia is promptly
reabsorbed,causing hyperammonemia and hepatic encephalopathy.
None of the other drugs causes hyperammonemia in cirrhotic patients.
Answer. a

Copyright 2014 Delhi Academy of Medical Sciences, All Rights Reserved. 5/88
(6). Serotonin (5-HT) is one of several mediators of the signs and symptoms of carcinoid tumor.When released from this neoplasm, most of
the manifestations of 5-HT are the result of activation of the 5-HT2 receptor. These manifestations are therefore likely
a. Bronchospasm
b. Constipation
c. Skeletal muscle weakness
d. Tachycardia

Solution. Ans-6: (a) Bronchospasm
Ref:Read the text below
Sol:
5-HT2 receptors mediate smooth muscle contraction and thus cause bronchospasm and diarrhea, not constipation.
Skeletal muscle and cardiac muscle are relatively insensitive to 5-HT.
Peptic ulceration is not mediated in part by 5-HT; the autacoid actually reduces acid secretion.
Answer. a

(7). Which of the following drugs is useful in asthma because it stabilizes mast cells?
a. Albuterol
b. Cromolyn
c. Ipratropium
d. Salmeterol

Solution. Ans-7: (b) Cromolyn
Ref:Read the text below
Sol:
Cromolyn is a prophylacticagent for asthma that appears to stabilize mast cells.
Albuterol is a shorter acting beta-2-selective agonist that is used for acute, not prophylactic, therapy.
Ipratropiumis an antimuscarinic drug.
Salmeterolhas a long duration beta-2-agonist action that appears to have an anti-inflammatory action and is useful in prophylaxis of
asthmatic attacks.
It has a slow onset of action that makes it useless in treating acute attacks.
Answer. b

Copyright 2014 Delhi Academy of Medical Sciences, All Rights Reserved. 6/88
(8). Which of the pairs is incorrect
a. V1-smooth muscles
b. V2-distal nephron
c. V3-anterior pituitary
d. V4-CNS

Solution. Ans 8: (d) V4-CNS
Ref:Read the text below
Sol:
Vasopressin Receptors
There are at least three kinds of vasopressin receptors: V1A, V1B, and V2.
All are G protein-coupled.
The V1A and V1B receptors act through phosphatidylinositol hydrolysis to increase the intracellular Ca2+ concentration.
The V2 receptors act through Gs to increase cyclic adenosine 3',5'-monophosphate (cAMP) levels.
The mechanism by which vasopressin exerts its antidiureticeffect is activated by V2 receptors and involves the insertion of
proteins called water channels into the apical (luminal) membranes of the principal cells of the collecting ducts.
Movement of water across membranes by simple diffusion is now known to be augmented by movement through water
channels called aquaporins, and to date 13 (AQP0AQP12) have been identified and water channels are now known to be
expressed in almost all tissues in the body.
The vasopressin-responsive water channel in the collecting ducts is aquaporin-2. These channels are stored in endosomes inside the
cells, and vasopressin causes their rapid translocation to the luminal membranes.
V1A receptors mediate the vasoconstrictor effect of vasopressin, and vasopressin is a potent stimulator of vascular smooth
muscle in vitro.
However, relatively large amounts of vasopressin areneeded to raise blood pressure in vivo, because vasopressin also acts on the
brain to cause a decrease in cardiac output. The site of this action is the area postrema, one of the circumventricular organs.
Hemorrhage is a potent stimulus for vasopressin secretion, and the blood pressure fall after hemorrhage is more marked
in animals that have been treated with synthetic peptides that block the pressor action of vasopressin.
V1A receptors are also found in the liver and the brain. Vasopressin causes glycogenolysis in the liver, and, as noted above, it is a
neurotransmitter in the brain and spinal cord.
The V1B receptors (also called V3 receptors) appear to be unique to the anterior pituitary, where they mediate increased
adrenocorticotropic hormone (ACTH) secretion from the corticotropes.
Answer. d

Copyright 2014 Delhi Academy of Medical Sciences, All Rights Reserved. 7/88
(9). Drug used in estrogen positive breast cancer
a. Tamoxifen
b. Cyproterone acetate
c. Flutamide
d. Fluoxetine

Solution. Ans 9: (a) Tamoxifen
Ref:Read the text below
Sol:
Adjuvant Regimens in breast cancer
In general, premenopausal women for whom any form of adjuvant systemic therapy is indicated should receive multidrug
chemotherapy.
The antiestrogen tamoxifen improves survival in premenopausal patients with positive estrogen receptors and should be added
following completion of chemotherapy.
Prophylactic castration may also be associated with a substantial survival benefit (primarily in estrogen receptorpositive
patients)
Approaches to Adjuvant Therapy
Age Group
Lymph Node
Status
Endocrine
Receptor (ER)
Status
Tumor Recommendation
Premenopausal Positive Any Any
Multidrug chemotherapy + tamoxifen if ER-positive +
trastuzumab in HER-2/neu positive tumors
Premenopausal Negative Any
>2 cm, or 12 cm with
other poor prognostic
variables
Multidrug chemotherapy + tamoxifen if ER-positive +
trastuzumab in HER-2/neu positive tumors
Postmenopausal Positive Negative Any
Multidrug chemotherapy + trastuzumab in HER-2/neu
positive tumors
Postmenopausal Positive Positive Any
Aromatase inhibitors and tamoxifen with or without
chemotherapy + trastuzumab in HER-2/neu positive tumors
Postmenopausal Negative Positive
>2 cm, or 12 cm with
other poor prognostic
variables
Aromatase inhibitors and tamoxifen + trastuzumab in HER-
2/neu positive tumors
Postmenopausal Negative Negative
>2 cm, or 12 cm with
other poor prognostic
variables
Consider multidrug chemotherapy + trastuzumab in HER-
2/neu positive tumors


The first decision is whether chemotherapy or endocrine therapy should be used.
While adjuvant tamoxifen improves survival regardless of axillary lymph node status, the improvement in survival is modest for
patients in whom multiple lymph nodes are involved.
For this reason, it has been usual to give chemotherapy to postmenopausal patients who have no medical contraindications and
who have more than one positive lymph node; tamoxifen is commonly given simultaneously or subsequently.
For postmenopausal women for whom systemic therapy is warranted but who have a more favorable prognosis, tamoxifen may be
used as a single agent. Large clinical trials have shown superiority for aromatase inhibitors over tamoxifen alone in the
adjuvant setting.
Other adjuvant treatments under investigation include the use of taxanes, such as paclitaxel and docetaxel, and therapy based on
alternative kinetic and biologic models. In such approaches, high doses of single agents are used separately in relatively
dose-intensive cycling regimens.
Answer. a

Copyright 2014 Delhi Academy of Medical Sciences, All Rights Reserved. 8/88
(10). SERM used for osteoporosis
a. Raloxifen
b. Estrogen
c. Cyproterone acetate
d. Fluoxetine

Solution. Ans 10: (a) Raloxifen
Ref:Read the text below
Sol:
Drugs Used to Treat Osteoporosis
Bisphosphonates
These analogs of pyrophosphate, including etidronate, risedronate, alendronate, ibandronate, pamidronate, tiludronate, and
zoledronic acid, comprise an important drug group used for the treatment of disorders of bone remodeling, such as
osteoporosis and Paget's disease, as well as for treatment of bone metastases and hypercalcemia of malignancy.
In addition, alendronate, risedronate,and ibandronate have been approved for the prevention and treatment of osteoporosis.
Zoledronic acidis also approved for the treatment of postmenopausal osteoporosis.
The bisphosphonates decrease osteoclastic bone resorption via several mechanisms, including
1) Inhibition of the osteoclastic proton pump necessary for dissolution of hydroxyapatite,
2) Decrease in osteoclastic formation/activation,
3) Increase in osteoclastic apoptosis (programmed cell death), and
4) Inhibition of the cholesterol biosynthetic pathway important for osteoclast function.
Selective estrogen-receptor modulators
Estrogen replacement is an effective therapy for the prevention of postmenopausal bone loss. When initiated in the immediate
postmenopausal period, estrogen therapy prevents osteoporosis and reduces the risk of hip fracture.
Raloxifeneis a selective estrogen-receptor modulator approved for the prevention and treatment of osteoporosis.
It increases bone density without increasing the risk of endometrial cancer. In addition, raloxifene may reduce the risk of invasive
breast cancer.
Raloxifeneis a first-line alternative for postmenopausal osteoporosis in women who are intolerant to bisphosphonates.
Raloxifene reduces serum total and low-density lipoprotein cholesterol concentrations.
The risk of venous thromboembolism appears to be comparable to that with estrogen. Other adverse effects include hot flashes and leg
cramps.
Calcitonin
Salmon calcitonin, administered intranasally, is effectiveand well tolerated in the treatment of postmenopausal osteoporosis.
The drug reduces bone resorption, but it is less effective than the bisphosphonates. A unique property of calcitonin is the relief of pain
associated with osteoporotic fracture.
Therefore, calcitonin may be beneficial in patients who have recently suffered a vertebral fracture
Teriparatide
Teriparatideis a recombinant segmentof human parathyroid hormone that is administered subcutaneously for the treatment of
osteoporosis.
Parathyroid hormone given continuously leads to dissolution of bone, but when it is given subcutaneously once daily, bone formation
is the predominant effect. It increases spinal bone density and decreases the risk of vertebral fracture.
Teriparatideis the first approved treatment for osteoporosis that stimulates bone formation.
Other drugs approved for this indication inhibit bone resorption.
It is also effective in the treatment of glucocorticoid-induced osteoporosis. Teriparatide has been associated with an
increased risk of osteosarcoma in rats.
Answer. a

Copyright 2014 Delhi Academy of Medical Sciences, All Rights Reserved. 9/88
(11). One of the many mechanisms of adverse events is its increased binding to secondary targets, usually proteins. With respect to the
primary target, the secondary target -
a. Should have a similar primary structure
b. Should have a absolutely identical secondary structure
c. Should have similar binding sites
d. Should be present in the same tissue as the primary target

Solution. Ans-11 (c) Should have similar binding sites
Ref: Read the text below
Sol:
A very common mechanism of adverse reactions caused by drugs is binding to non-target proteins.
These non-target proteins may be receptors or enzymes located in different organs as compared to the target organ (option 4)
(For e.g. Opioids act on receptors in CNS for therapeutic effects, but effects on receptors in GIT can cause constipation as a side effect).
The essence of these non-target proteins is that they share binding sites for the drug with the target protein of the drug (option
3).
This binding site is a function of the tertiary structure of the protein.
Hence, proteins with different primary and secondary structures (option 1 and 2) but with similar binding sites or similar
tertiary structures can also act as non-target proteins (e.g. V
1
and V
2
receptors have different structures but share the same binding site;
hence, both are acted upon by desmopressin).
Answer. c

(12). Which of the following is known as 'suicide enzyme'?
a. 5-Lipoxygenase
b. Cyclo-oxygenase
c. 5'-nucleotidase
d. Thromboxane synthase

Solution. Ans 12: (b) Cyclo-oxygenase
Ref Read the text below
Sol:
: Some clinical examples of suicide inhibitors include:
Aspirin, which inhibits cyclooxygenase 1 and 2 enzymes.
Penicillin, which inhibits DD-transpeptidase from building bacterial cell walls.
Sulbactam, which prohibits penicillin-resistant strains of bacteria from metabolizing penicillin.
Allopurinol, which inhibits uric acid production by xanthine oxidase in the treatment of gout.
AZT(zidovudine) and other chain-terminating nucleosideanalogues used to inhibit HIV-1reverse transcriptasein the
treatment of HIV/AIDS.
Eflornithine, one of the drugs used to treat sleeping sickness, is a suicide inhibitor of ornithine decarboxylase.
Sarin is a suicide inhibitor of acetylcholinesterase.
5-fluorouracil acts as a suicide inhibitor of thymidylate synthase during the synthesis of thymine from uridine. This reaction is crucial
for the proliferation of cells, particularly those that are rapidly proliferating (such as fast-growing cancer tumors). By inhibiting this step,
cells die from a thymineless death because they have no thymine to create more DNA. This is often used in combination with
Methotrexate, a potent inhibitor of dihydrofolate reductase enzyme.
Exemestane, a drug used in the treatment of breast cancer, is an inhibitor of the aromatase enzyme.
Selegiline,
[1]
although in the attached reference the compound is called a 'suicide inactivator' (not inhibitor).
Answer. b

Copyright 2014 Delhi Academy of Medical Sciences, All Rights Reserved. 10/88
(13). Which of the following statements describes the major difference between zolpidem and older hypnotics such as barbiturates and
benzodiazepines?
a. Associated with thrombocytopenia
b. Does not act on chloride channels in the CNS
c. Has significant anticonvulsant activity
d. Reduced addiction liability

Solution. Ans-13: (d) Reduced addiction liability
Ref:Read the text below
Sol:
Zolpidem has greatly reduced addiction liability compared to barbiturates and benzodiazepines even though it does act on
chloride channels.
Zolpidem does not havemajor anticonvulsant actions, a longer duration of action, or thrombocytopenic toxicity.
Answer. d

(14). Which of the following drugs is used in cerebral palsy to reduce skeletal muscle spasticity by an action on cholinergic nerve endings?
a. Baclofen
b. Botulinum toxin
c. Dantrolene
d. Diazepam

Solution. Ans-14: (b) Botulinum toxin
Ref:Read the text below
Sol:
Botulinum toxin actson cholinergic nerve endings (including skeletal muscle motor nerve endings) to reduce acetylcholine release.
Baclofen acts in the CNS to activate GABAB receptors; some sedation may result.
Dantroleneacts within the skeletal muscle cell to reduce calcium release from the sarcoplasmic reticulum; reduction of muscle spasm
results without central nervous system depression.
Diazepam facilitates GABAergic transmission in the brain and spinal cord; it does not act on cholinergic transmission.
Answer. b

(15). Which of the following drugs is most likely to cause parkinsonian adverse effects?
a. Clozapine
b. Fluphenazine
c. Haloperidol
d. Olanzapine

Solution. Ans-15: (c) Haloperidol
Ref:Read the text below
Sol:
Haloperidol is a verypotent and efficacious antipsychotic but also manifests the highest incidence of extrapyramidal effects.
Clozapine, olanzapine, and quetiapineare newer, atypical antipsychotic agents with much lower incidence of extrapyramidal
toxicity.
Answer. c

Copyright 2014 Delhi Academy of Medical Sciences, All Rights Reserved. 11/88
(16). Apatient has been treated for bipolar disorder for several months. Which of the following is a common adverse effect of lithium
carbonate?
a. Dry mouth
b. Hyperthyroidism
c. Leukopenia
d. Nephrogenic diabetes insipidus

Solution. Ans-16: (d) Nephrogenic diabetes insipidus
Ref:Read the text below
Sol:
Nephrogenic diabetes insipidus is a common adverse effect of lithium at therapeutic dosage.
Dry mouth and other anticholinergic toxicities are common with tricyclic antidepressants and some phenothiazines.
Lithium may induce hypothyroidism, but not hyperthyroidism
Similarly, lithium increases rather than decreases white blood cell count
Answer. d

(17). Methadone has been used in opioid addiction maintenance clinics with partial success. Its ability to reduce the use of heroin and
morphine by recovering addicts can be ascribed to which of the following mechanisms?
a. Agonist effects at central dopamine d2 receptors
b. Antidepressant effects at serotonin (5-ht) receptors
c. Antagonist actions at the mu opioid receptor
d. Long-lasting saturation of the mu opioid receptor and development of tolerance

Solution. Ans-17: (d) Long-lasting saturation of the mu opioid receptor and development of tolerance
Ref:Read the text below
Sol:
Methadone is a typical strong agonist at opioid mu receptors and differs from morphine chiefly in its long duration of binding to
the receptor and long elimination half-life.
It therefore produces a prolonged opioid effect that causes tolerance to the euphoric action of short-acting heroin and
morphine.
Methadone has no opioid antagonist action nor does it act on metabolic pathways, serotonin receptors, or dopamine receptors).
Answer. d

(18). Which of the following drugs has opioid antagonist actions and also reduces nicotine and alcohol craving in persons dependent on those
drugs?
a. Bupropion
b. Flumazenil
c. Nalbuphine
d. Naltrexone

Solution. Ans-18: (d) Naltrexone
Ref:Read the text below
Sol:
Naltrexone is a very long-acting opioid antagonistthat has also been shown to reduce craving and dependence in smokers and
alcoholics.
The mechanisms for the latter effects are not fully understood.
Bupropion is an antidepressantthat has some efficacy in tobacco dependence. It is not an opioid antagonist.
Flumazenil is a benzodiazepineantagonist that is useful in BDZ overdose. It has no opioid antagonist effect.
Nalbuphine is an opioid partial agonistthat has analgesic effects but can also antagonize the actions of strong agonists like
morphine.
Answer. d

Copyright 2014 Delhi Academy of Medical Sciences, All Rights Reserved. 12/88
(19). A known drug abuser is in restraints, combative, and hallucinating. His blood pressure, heart rate, and body temperature are
elevated.Pupils are dilated and demonstrate both horizontal and vertical nystagmus. Which of the following drugs would cause th
a. Amphetamine
b. Heroin
c. Phencyclidine
d. Scopolamine

Solution. Ans-19: (c) Phencyclidine
Ref:Read the text below
Sol:
Phencyclidine, an NMDA receptor antagonist, is a more violent hallucinogen than most other drugs in this class (LSD, mescaline,
scopolamine).
It causes sympathetic discharge and muscle twitching; nystagmus is very common.
Amphetamine is a sympathomimetic stimulant that can produce hallucinations or toxic psychosis in overdose, but is not usually
associated with nystagmus.
Heroin is a depressant drug and does not present with combative stimulation. Pupils are constricted by opioids
Scopolamine similarly does not usually cause combativeness or nystagmus.
Answer. c

(20). Which of the following antifungal drugs is strongly associated with drug interactions resulting from inhibition of CYP 3A4?
a. Amphotericin B
b. Caspofungin
c. Fluconazole
d. Ketoconazole

Solution. Ans-20: (d) Ketoconazole
Ref:Read the text below
Sol:
Ketoconazole is a potent inhibitor of the cytochrome P450 enzyme, CYP 3A4.
Because CYP 3A4 is involvedin the metabolism of almost 50% of the drugs in use, ketoconazole is implicated in many
pharmacokinetic drug interactions.
Amphotericin B causes nephrotoxicityand may therefore interfere with the excretion of other drugs.
Caspofunginappears to be free of most drug interactions.
Fluconazole although a member of the same (azole) group as ketoconazole, is much less potent as a CYP 3A4 inhibitor and is
associated with far fewer drug interactions.
Answer. d

Copyright 2014 Delhi Academy of Medical Sciences, All Rights Reserved. 13/88
(21). Which of the following injections is available for subcutaneous administration?
a. Albuterol
b. Terbutaline
c. Metaproterenol
d. Pirbuterol

Solution. Ans 21: (b) Terbutaline
Ref Read the text below
Sol:
Terbutaline Sulfate Injection, USP is indicated for the prevention and reversal of bronchospasm in patients 12 years of age and
older with asthma and reversible bronchospasm associated with bronchitis and emphysema.
Contraindications
1. Prolonged Tocolysis
Terbutaline sulfate has not been approved and should not be used for prolonged tocolysis (beyond 48-72 hours). In particular,
Terbutaline sulfate should not be used for maintenance tocolysis in the outpatient or home setting
2. Hypersensitivity
Terbutaline sulfate injection is contraindicated in patients known to be hypersensitive to sympathomimetic amines or any component of
this drug product.
Answer. b

(22). A drug being developed as an antiarrhythmic agent was studied in the laboratory using microelectrode techniques for measuring the
transmembrane potential. The results of this study are shown in Fig. Which of the following standard antiarrhythmic agents does the new
drug most resemble?

a. Adenosine
b. Ibutilide
c. Lidocaine
d. Procainamide

Solution. Ans-22: (b) Ibutilide
Ref: Read the text below
Sol :
The recorded transmembrane potential indicates that the new drug has no effect on the upstroke velocity but prolongs the action
potential duration of the ventricular muscle fiber.
This effect is characteristic of blocking potassium (IK) channels that open during repolarization, actions shown by group III
antiarrhythmic drugs such as ibutilide. Adenosine hyperpolarizes AV nodal cells and blocks propagation of impulses through this part
of the heart.
It does not prolong the action potential. Lidocaine is a group IB drug and slows upstroke velocity of the action potential in susceptible
cells but shortens, rather than prolongs, the action potential duration.
Procainamide is a group IA drug that slows upstroke velocityas well as prolonging the action potential.
Answer. b

Copyright 2014 Delhi Academy of Medical Sciences, All Rights Reserved. 14/88
(23). Time dependent killing and prolonged post-antibiotic effect is seen with -
a. Fluoroquinolones
b. Beta lactam antibiotics
c. Clindamycin
d. Erythromycin

Solution. Ans 23: (a) Fluoroquinolones
Ref Read the text below
Sol:
With Concentration Dependant Killing (CDK)
Aminoglycoside
Fluroquinolones
Metronidazole
Chloramphenicol
Streptogramins
With TIME Dependant Killing (TDK) (penicillins, cephalosporins, carbapenems, monobactams), clindamycin, macrolides
(erythromycin, clarithromycin), oxazolidinones (linezolid)
Beta-lactam
Vancomycin
Macrolide
Clindamycin
POST ANTIBIOTIC EFFECT PAE
All antimicrobial---- Gm+ve Org ONLY
Aminoglycoside
Quinolone
Rifampicin-----Gm-ve also
Carbapenem
Azithro&Clarithro
PAEs of the fluoroquinolones (ciprofloxacin, ofloxacin, levofloxacin, moxifloxacin and
garenoxacin) were 25 h. The macrolide (erythromycin, clarithromycin and telithromycin) PAEs were 14 h,and that of
clindamycin was 2 h. The PAEs induced by tetracycline and minocycline were 13 h. The PAEs induced by the -lactams
(penicillin G, amoxicillin and ceftriaxone), vancomycin, linezolid and chloramphenicol were 12 h. The PAE induced by
rifampicin was 45 h. Quinupristin/dalfopristin had the longest PAE, lasting
for 78 h.
Answer. a

(24). Which of the following opioid receptors is responsible for dysphoric effects?
a. Mu
b. Kappa
c. Delta
d. Sigma

Solution. Ans 24: (b) Kappa
Ref Read the text below
Sol:
Mu and delta receptor-
P- Physical dependence
M- Miosis
C- Constipation
A- Analgesia
R- Respiratory depression
E- Euphoria
S- Sedation

Kappa receptor-
D- Dysphoria
M- Miosis
A- Analgesia
R- Respiratory depre
D- Diuresis
S- Sedation
Answer. b
Copyright 2014 Delhi Academy of Medical Sciences, All Rights Reserved. 15/88

(25). A drug 'X' belongs to the anticholinergic drug group. It is primarily used in pre anesthetic medication and also during surgery. Which of
the following can be 'X' ?
a. Glycopyrrolate
b. Pipenzolate methyl bromide
c. Isopropamide
d. Dicyclomine

Solution. Ans-25: (a) Glycopyrrolate
Ref: KDTs - 110
Sol :
Glycopyrrolate is used to reducethe secretions (to prevent reflex bronchospasm) during anaesthesia.
In anesthesia, glycopyrrolate injection can be used as a preoperative medication in order to reduce salivary, tracheobronchial,
and pharyngeal secretions, as well as decreasing the acidity of gastric secretion.
It is also used in conjunction with neostigmine, a neuromuscular blocking reversal agent, to prevent neostigmine's muscarinic effects
such as bradycardia.
Answer. a

(26). All of the following mydriatics produce cycloplegia except
a. Phenylephrine
b. Tropicamide
c. Cyclopentolate
d. Homatropine

Solution. Ans-26: (a) Phenylephrine
Ref: KDTs - 113, 114
Sol :
Anticholinergic agentsproduce both cycloplegia as well as mydriasis whereas sympathomimetics cause mydriasis without
inducing cycloplegia.
Answer. a

(27). Which of the following drugs shows the phenomenon of vasomotor reversal of Dale after administration of an adrenergic blocker ?
a. Adrenaline
b. Noradrenaline
c. Isoprenaline
d. All of the above

Solution. Ans-27: (a) Adrenaline
Ref: KDTs - 123
Sol :
Vasomotor reversal of dale is seen with adrenaline.
When this drug is infused quickly, initially there is rise in blood pressure (due to sitmulation of receptors) followed by
prolonged fall (
2
action). blocking drugs inhibit the initial rise and only fall in blood pressure is recorded.
This is known as vasomotor reversal.
Nor adrenaline has no
2
activity and isoprenaline lacks activity, therefore cannot demonstrate this phenomenon.
Answer. a

Copyright 2014 Delhi Academy of Medical Sciences, All Rights Reserved. 16/88
(28). A drug has a volume of distribution of 40 L, half life of 1 day, the total body clearance of the drug is :
a. 8 ml/min
b. 10 ml/min
c. 19 ml/min
d. 29 ml/min

Solution. Ans-28: (c) 19 ml/min
Ref: Read the text below
Sol :
Clearance, theoretically, is the volume of plasma that is cleared off all drugs in a unit time.

Answer. c

(29). Which of the following is a prodrug ?
a. Methyl-dopa
b. Captoril
c. Atropine
d. Adrenaline

Solution. Ans-29: (a) Methyl-dopa
Ref: Read the text below
Sol :
-Methyldopa is a prodrugi.e. inactive as such but gets converted in the body into an active metabolite, a-methyl nor-epinephrine.
Answer. a

Copyright 2014 Delhi Academy of Medical Sciences, All Rights Reserved. 17/88
(30). Vasopressin antagonists act on which part of the nephron?
a. Proximal convoluted tubule
b. Distal convoluted tubule
c. Cortical collecting tubule
d. Medullary collecting duct

Solution. Ans 30: (d) Medullary collecting duct
Ref Read the text below
Sol:
A vasopressin receptor antagonist (VRA) is an agent which interferes with action at the vasopressin receptors. Most commonly
VRAs are used in the treatment of hyponatremia, especially in patients with congestive heart failure, liver cirrhosis or SIADH
Vasopressin has two effects by which it contributes to increased urine osmolarity (increased concentration) and decreased water
excretion. These are:
1.Increasing the water permeability of distal tubule and collecting duct cells in the kidney, thus allowing water reabsorption and
excretion of more concentrated urine, i.e., antidiuresis. in
2.Increasing permeability of the inner medullary portion of the collecting duct to urea by regulating the cell surface expression of urea
transporters,
[4]
which facilitates its reabsorption into the medullary interstitium as it travels down the concentration gradient created by
removing water from the connecting tubule, cortical collecting duct, and outer medullary collecting duct.
Answer. d

(31). Which of the following is an enzyme inducer ?
a. Erythromycin
b. Pyrimethamine
c. Pyrizinamide
d. Rifampicin

Solution. Ans-31: (d) Rifampicin
Ref: Read the text below
Sol :
Pentamidine, an antimicrobial agent, is not a hepatic enzyme inducer.
Antiepileptics (Phenobarbitone, Phenytoin, Primidone), antituberculars
(rifampicin, INH), Phenylbutazone
and chr. Alcoholism and smoking are potent enzyme inducers.
Answer. d

(32). Pharmacogenetics is important in the metabolism of
a. Rifampicin
b. Isoniazid
c. Digitalis
d. Propranolol

Solution. Ans-32: (b) Isoniazid
Ref: Read the text below
Sol :
Pharmacogenetics is important in the metabolism of isoniazid (INH).
INH is mainly metabolized by acetylation. The rate of acetylation shows genetic variation.
There are either.
Fast acetylators (30 40% of Indians) or
Slow acetylators (60 70% of Indians).
Answer. b

Copyright 2014 Delhi Academy of Medical Sciences, All Rights Reserved. 18/88
(33). Which of the following is wrong regarding treatment with iodine?
a. It causes acute inhibition of synthesis of diodotyrosine and diodothyronine
b. It also inhibits the release of thyroid hormones
c. Its use is contraindicated in hyperthyroidism
d. It causes iodism

Solution. Ans 33: (c) Its use is contraindicated in hyperthyroidism
Ref Read the text below
Sol:
Potassium iodideis usually used to treat acute thyrotoxicosis, usually as a saturated solution of potassium iodide called SSKI. It is
also used to block uptake of iodine-131 in the thyroid gland (see isotopes section above), when this isotope is used as part of
radiopharmaceuticals (such as iobenguane) that are not targeted to the thyroid or thyroid-type tissues.
It causes acute inhibition of synthesis of diodotyrosine and diodothyronine
It also inhibits the release of thyroid hormones
Answer. c

(34). Which of the following is useful in rheumatoid arthritis because it binds tumor necrosis factoralpha (TNF-alpha)?
a. Leflunomide
b. Infliximab
c. Methotrexate
d. Paclitaxel

Solution. Ans-34: (b) Infliximab
Ref:Read the text below
Sol:
Infliximab is a chimeric monoclonal antibodythat binds TNF-alpha and is useful in rheumatoid arthritis, ulcerative colitis, and
several other immune diseases.
Leflunomide inhibits ribonucleotidesynthesis and thereby reduces T-cell proliferation.
Methotrexate is an antimetabolitethat inhibits cell proliferation. It is useful in rheumatoid arthritis.
Paclitaxel is a microtubule-binding agent that is cytotoxic in certain neoplasms and slows restenosis in coronarystents. It is
not used in autoimmune diseases.
Answer. b

(35). Which of the following agents used to prevent organ transplant rejection is most likely to cause significant decrease in creatinine
clearance?
a. Antilymphocyte globulin
b. Cyclosporine
c. Mycophenolate mofetil
d. Prednisone

Solution. Ans-35: (b) Cyclosporine
Ref:Read the text below
Sol:
Cyclosporine and tacrolimus are associated with nephrotoxicity as a major adverse effect.
Antilymphocyte globulin is associated with foreign protein reactions, including anaphylaxis.
Mycophenolate mofetil causes GI disturbances and myelosuppression.
Prednisone causes a variety of chronic toxicities conveniently summarized as Cushingoid effects but has little acute toxicity.
Answer. b

Copyright 2014 Delhi Academy of Medical Sciences, All Rights Reserved. 19/88
(36). Which of the following antidiabetic agents acts on the peroxisome proliferator-activated receptor-gamma (PPAR-gamma) nuclear
receptor?
a. Acarbose
b. Glyburide
c. Insulin lispro
d. Pioglitazone

Solution. Ans-36: (d) Pioglitazone
Ref:Read the text below
Sol:
The glitazone drugs (pioglitazone, rosiglitazone) reduce insulin resistance in peripheral tissues by activating the PPAR-gamma
receptor, which promotes glucose uptake and utilization.
Acarbose interferes with the action of intestinal alpha-glucosidases and reduces absorption of glucose.
Glyburide and other sulfonylurea hypoglycemic drugs block the ATP-activated potassium channel in pancreatic cells and cause
increased insulin release.
Insulin lispro is a modified insulin with rapid onset and offset of action. All insulins act by causing insertion of glucose transporters
into cell membranes and several other mechanisms.
The mechanism of action of metformin and similar biguanide antidiabetic drugs is still unclear but may involve reduction of
glucagon release, stimulation of glycolysis in peripheral tissues, or decreased hepatic and renal gluconeogenesis.
Answer. d

(37). Which of the following mechanisms most accurately describes the blocking action of methimazole?
a. Iodine uptake by the thyroid
b. Organification of iodine in the thyroid
c. Peripheral conversion of T4 to T3
d. Peripheral thyroxine (T4) and T3 receptors

Solution. Ans-37: (b) Organification of iodine in the thyroid
Ref:Read the text below
Sol:
Methimazole (and other thioamides) act primarilyby inhibiting thyroid peroxidase, the critical enzyme in iodine organification.
They also inhibit coupling of iodotyrosines in thyroglobulin.
They do not block iodine uptake by the gland. Block of iodine uptake by the thyroid is a property of antithyroid anions such as
thiocyanate (SCN) and perchlorate (ClO4 ).
Inhibition of peripheral conversion of T4 to T3 is the mechanism of beta blockers and iodine-containing radio-opaque
drugs such as ipodate.
Block of peripheral thyroid receptors is not a mechanism of any currently available drug.
Answer. b

Copyright 2014 Delhi Academy of Medical Sciences, All Rights Reserved. 20/88
(38). The increased risk of blood clots in women who use certain combined estrogen/progestin birth control pills is well established. Which of
the following best explains this association?
a. Impaired glucose tolerance
b. Increased Factor II, VII, IX, and X levels produced by estrogens
c. Increased plasminogen levels produced by progestins
d. Increased thyroxine-binding globulin

Solution. Ans-38: (b) Increased Factor II, VII, IX, and X levels produced by estrogens
Ref:Read the text below
Sol:
Increased clotting factor synthesis (and decreased antithrombin III synthesis) caused by estrogens are the major factors
responsible for increased incidence of thromboembolic phenomena. Glucose tolerance is impaired in some women by progestins and
would not be expected to affect clotting.
Plasminogen is increased by estrogens, not progestins, but would decrease clotting.
Thyroxine-binding globulin is also increased by estrogen, but would have little effect on clotting.
Triglycerides are increased by estrogens and would have little or no effect on clotting.
Answer. b

(39). Which of the following drugs is considered afirst-line agent in the chronic treatment of rheumatoid arthritis and in the treatment of
choriocarcinoma?
a. Cyclosporine
b. Hydroxychloroquine
c. Methotrexate
d. Sulfasalazine

Solution. Ans-39: (c) Methotrexate
Ref:Read the text below
Sol:
Methotrexate in very low dosagehas proven effective in slowing the progression of rheumatoid arthritis and has low toxicity.
It is also one of the most effective (in much higher dosage) antimetabolite cancer chemotherapeutic drugs used in
choriocarcinoma.
Cyclosporine is not used in either condition but is a first-line drug in transplant immunopharmacology.
Hydroxychloroquine is a disease-modifying antirheumatoid arthritis drug, but has no efficacy in the treatment of cancer. It is more
toxic and less effective in rheumatoid arthritis than methotrexate.
Sulfasalazine is also effective in slowing the progression of joint damage in rheumatoid arthritis but has no efficacy in
cancer.
Answer. c

(40). Which of the following laxatives acts osmotically to increase the water content of the stool?
a. Cascara
b. Docusate
c. Glycerine
d. Magnesium hydroxide

Solution. Ans-40: (d) Magnesium hydroxide
Ref:Read the text below
Sol:
Magnesium hydroxideis an osmotic laxative that retains water in the lumen of the colon.
Cascara is a naturally occurring plant product that stimulates the bowel and causes evacuation within 612 hours when given orally.
Docusate is a stool-softeningagentthat acts entirely within the lumen.
Glycerine given by suppository, is a stool softener.
Answer. d

Copyright 2014 Delhi Academy of Medical Sciences, All Rights Reserved. 21/88
(41). Which of the following drugs binds to, and prevents disassembly of, microtubules?
a. Busulfan
b. Cisplatin
c. Mercaptopurine
d. Paclitaxel

Solution. Ans-41: (d) Paclitaxel
Ref:Read the text below
Sol:
Paclitaxel is a plant alkaloidthat interferes with microtubule disassembly.
Busulfan and cisplatin are alkylatingagents that interfere with DNAmobilization and replication.
Mercaptopurine is an antimetabolitethat interferes with purine synthesis.
Vinblastine and vincristineand vinorelbine are mitotic poisons that prevent cancer cell division by interfering with microtubule
assembly and thus immobilizing the mitotic spindle apparatus.
Their indications and toxicities are quite different from those of paclitaxel
Answer. d

(42). Which of the following drugs blocks the action of angiotensin II at the AT1 receptor?
a. Bradykinin
b. Enalapril
c. Losartan
d. Ondansetron

Solution. Ans-42: (c) Losartan
Ref:Read the text below
Sol:
Losartan and severalother drugs block the action of angiotensin II at its primary cardiovascular target, the AT1 receptor.
They are therefore useful in hypertension and heart failure. Bradykinin (choice A) is a vasodilator peptide.
It is not therapeutically useful because of undesirable side effects.
Enalapril is an ACE inhibitorand reduces the production of angiotensin II from angiotensin I.
Ondansetron is a 5-HT3 receptorantagonist used to prevent chemotherapy-induced and postoperative vomiting.
Answer. c

(43). Resistance to which of the following drugsoccurs when the bacterium expresses higher quantities of a drug-efflux pump?
a. Amoxicillin
b. Clarithromycin
c. Doxycycline
d. Gentamicin

Solution. Ans-43: (c) Doxycycline
Ref:Read the text below
Sol:
Resistance to tetracyclinessuch as doxycycline is associated with production of an efflux pump or alteration of the 30S ribosomal-
binding site.
Resistance to beta lactams such as amoxicillinusually results from production of beta lactamase.
Macrolides such as clarithromycinerythromycin, and azithromycin bind to the 50S ribosome in susceptible organisms and prevent
protein synthesis.
Simple methylation of the binding site prevents the bacteriostatic action of these drugs and many bacteria have such mutation.
Resistance to aminoglycosidessuch as gentamicin is usually due to synthesis of inactivating enzymes by the bacteria.
Answer. c

Copyright 2014 Delhi Academy of Medical Sciences, All Rights Reserved. 22/88
(44). Which of the following trials can be performed to determine the maximal tolerated dose of a drug?
a. Case control
b. Phase II - RCT
c. Phase I
d. Phase VI - RCT

Solution. Ans 44: (c) Phase I
Ref Read the text below
Sol:
Maximum Tolerated Dose (MTD) Highest dosage of a drug, drug combination or other treatment modality that patients can safely
tolerate. Usually determined by Phase I Trial.

PHASE I CLINICAL TRIAL
Initial exposure of human to investigational drug
Healthy male volunteers
Single site by clinical pharmacologist
Small no ( 2025)
Mainly for toxicity & PK, not for efficacy
Non blinding / Open
Begin with 1/5 or 1/10
th
of maximum tolerable dose in animals
Phase I trail for anticancer done in pts

PHASE II
Early phase
20200
Use / side effects
To establish a dose range
Single blind (subject)
Late phase
50300
Double blind (randomized/controlled)
Safety & efficacy specific disease
compare with existing drug

PHASE III CLINICAL TRIAL
250-1000 patients
Confirmation of effective doses
Safety / efficacy
Minimize errors of phase I & II
Multicentric
Double blind
Cross over
More than 5 yrs
Statistical evaluation

PHASE IV CLINICAL TRIAL
Post marketing surveillance
No fixed duration
Exposure of drugs to a wider population
Different formulation/ dosage forms/ duration/ drug interaction/ drug comparison studies
Detection of unknown ADR / Side effects
Phase 0-
Microdosing studies
Answer. c

Copyright 2014 Delhi Academy of Medical Sciences, All Rights Reserved. 23/88
(45). Ritonavir inhibits all of the following except -
a. Amiodarone
b. Phenytoin
c. Cisapride
d. Midazolam

Solution. Ans 45: (b) Phenytoin
Ref Read the text below
Sol:
The proposed mechanism involves ritonavir induction of phenytoin metabolism via CYP450 2C9
Titonavir is the mostpotent enzyme inducer of CYP3A4, so it it inhibits the metabolism of Amiodarone Cisapride Midazolam
Answer. b

(46). One of the concerns of drug designing is to -
a. Decrease the binding affinity of the drug to target protein
b. Increase the binding affinity of the drug to non-target proteins
c. Increase the number of interactions of the drug with the target protein
d. Decrease the potency of the drug

Solution. Ans 46: (c) Increase the number of interactions of the drug with the target protein
Ref Read the text below
Sol:
Active site identification is the first step in drug designing program.
It analyzes the protein to find the binding pocket, derives key interaction sites within the binding pocket, and then prepares the
necessary data for Ligand fragment link.
The basic inputs for this step are the 3D structure of the protein and a pre-docked ligand in PDB format, as well as their atomic
properties
Answer. c

(47). Epinephrine causes hyperglycemia by
a. Increased glucagon
b. Decreased insulin secretion
c. Increased glucocorticoids
d. Increased thyroxine

Solution. Ans-47: (a) Increased glucagon
Ref: KDTs - 124
Sol:
Epinephrine from the adrenal medulla acts via beta-adrenergic receptors, whereas norepinpherine is released from nerve
endings and acts on alpha2-adrenergic receptors.
Norepinephrine and epinephrine have somewhat opposing effects on insulin release (norepinephrine inhibits, epinephrine
stimulates), but the net effect of both is increased blood glucose.
This occurs via stimulation of glycogenolysis and release of glucose from hepatocytes (epinephrine), and indirectly through inhibition of
insulin release (norepinephrine), and release of growth hormone (epinephrine) and ACTH (which increases cortisol).
Answer. a

Copyright 2014 Delhi Academy of Medical Sciences, All Rights Reserved. 24/88
(48). All of the following effects are seen with cholinergic muscarinic receptor stimulation except
a. Sweating
b. Rise in blood pressure
c. Bradycardia
d. Urination

Solution. Ans-48 (b) Rise in blood pressure
Ref: KDT - 129
Sol:
Stimulation of muscarinic receptors decreases blood pressure..

Comparison of types
Type Function
M
1
EPSP in autonomic ganglia
secretion from salivary glands and stomach
In CNS (memory?)
[8]
M
2
slow heart rate
reduce contractile forces of atrium
reduce conduction velocity of AV node
In CNS
homotropic inhibition
M
3
smooth muscle contraction
Increase intracellular calcium in vascular endothelium
increased endocrine and exocrine gland secretions, e.g. salivary glands and stomach
In CNS
Eye accommodation
vasodilation
induce emesis
M
4
Activation of M4 causes decreased locomotion
[8]
In CNS
M
5
In CNS
Answer. b

(49). Which of the following antitubercular agents is a strong inducer of hepatic cytochrome P450?
a. Ethambutol
b. Isoniazid
c. Rifampin
d. Streptomycin

Solution. Ans-49: (c) Rifampin
Ref:Read the text below
Sol:
Rifampin selectivelyinhibits bacterial DNA-dependent RNA polymerase. It is very useful in treating mycobacterial infections
since it can penetrate cells and kill intracellular organisms.
It is one of the most potent inducers of cytochrome P450 known, leading to increased hepatic clearance of many other drugs
including the oral anticoagulants, cyclosporine, propranolol, digitoxin, corticosteroids, and oral contraceptives.
Ethambutol is often combined with isoniazid in antitubercular regimens. Clearance is primarily via renal excretion.
Isoniazid is the mostwidely used antitubercular agent. It functions by inhibiting mycolic acid biosynthesis.
Answer. c

Copyright 2014 Delhi Academy of Medical Sciences, All Rights Reserved. 25/88
(50). A 35-year-old woman with hypertension is planning to become pregnant. Which of the following is contraindicated in pregnancy?
a. Clonidine
b. Hydralazine
c. Hydrochlorothiazide
d. Losartan

Solution. Ans-50: (d) Losartan
Ref:Read the text below
Sol:
Losartan causes renal damagein the fetus, and renal impairment in renovascular disease. It is contraindicated in pregnancy.
Clonidine causessome sedation and rebound hypertension when stopped suddenly, but is not contraindicated in pregnancy.
Hydralazine causes a reversibletype of lupus erythematosus.
Hydrochlorothiazide may cause hypokalemia, dilutional hyponatremia, elevated lipids, hyperuricemia, and glucose
intolerance.
Answer. d

(51). A35-year-old opera singer has been treated for hypertension with enalapril. Although his blood pressure has been reduced, he complains
that he is now unable to perform because of a dry cough. Which of the following acts by a similar but not identical mechanism and is
much less likely to cause cough?
a. Captopril
b. Clonidine
c. Losartan
d. Prazosin

Solution. Ans-51: (c) Losartan
Ref:Read the text below
Sol:
Losartan, an AT1 receptor-blocking agent, has effects in hypertension similar to those of ACE inhibitors but causes a much lower
incidence of cough.
Captopril and other ACE inhibitorscause a dry cough in 520% of patients. This may be due to accumulation of bradykinin as a
result of ACE inhibition; ACE also metabolizes bradykinin. The cough disappears with cessation of ACE inhibitor treatment.
Pretreatment with aspirin reduces its frequency and severity in some patients.
Clonidine the centrally acting alpha- 2-receptor agonist, produces sedation and xerostomia but not cough.
The alpha-1-receptor antagonist prazosin produces postural hypotension but not cough.
The betablocker propranolol may produce a variety of side effects including precipitating heart failure and asthma in susceptible
patients.
It does not cause cough. Propranolol has been used by musicians to control palpitations associated with stage fright.
Answer. c

(52). Which of the following is the muscle relaxant of choice in renal failure ?
a. Rapacurium
b. Pancuronium
c. Atracurium
d. Rocuronium

Solution. Ans-52: (c) Atracurium
Ref:Read the text below
Sol:
Atracurium
The unique feature of atracurium is inactivation in plasma by spontaneous non enzymatic degradation (Hofmann elimination) in
addition to that by cholinesterases.
Consequently its duration of action is not altered in patients with hepatic / renal insufficiency or hypodynamic circulation.
Hemodynamically it is almost neutral, though histamine release may occur.
Therefore it is the agent of choice in renal failure
Answer. c
Copyright 2014 Delhi Academy of Medical Sciences, All Rights Reserved. 26/88

(53). The mydriatic having the fastest and shortest action is
a. Atropine
b. Homatropine
c. Tropicamide
d. Cyclopentolate

Solution. Ans-53: (c) Tropicamide
Ref: KDT 111
Sol:
Tropicamide is an antimuscarinic drug that produces short acting mydriasis (dilation of the pupil) and cycloplegia when applied as
eye drops. It is used to allow better examination of the lens, vitreous humor, and retina.
Due to its relatively short duration of effect (48 hours), it is typically used during eye examinationssuch as the dilated
fundus examination, but it may also be used before or after eye surgery.
Cycloplegic drops are often also used to treat anterior uveitis, decreasing risk of posterior synechiae and decreasing inflammation in
the anterior chamber of the eye
Answer. c

(54). What is the most dangerous effect of belladonna in very young children?
a. Dehydration
b. Hallucination
c. Hypertension
d. Hyperthermia

Solution. Ans-54: (d) Hyperthermia
Ref: KDTs - 113
Sol:
Atropa belladonna contains anticholinergic principles like atropine and hyoscine.
Atropine antagonises central muscarinic receptors (remember, the parasympathetic nervous system is made up of muscarinic and
nicotinic receptors). This muscarinic receptor antagonising alters temperature regulation in the hypothalamus the mechanism how
it works is unfortunately not understood. But is is known that it reduces blood flow to the skin, thus reducing the ability for the
body to lose heat and increasing body temperature
Atropine is contra-indicatedin children due to the risk of hyperthermia.
Answer. d

(55). Second generation anti-histaminics used in allergic rhinitis are all of the following except
a. Azelastine
b. Fexofenadine
c. Chlorpheniramine maleate
d. Desloratidine

Solution. Ans-55: (c) Chlorpheniramine maleate
Ref:read the text below
Sol:
The second generation antihistaminics are non sedative.
These include.
Terfenadine
Astemizole
Loratidine
Cetirizine
Azelastine
Fexofenadine
Desloratidine
Answer. c
Copyright 2014 Delhi Academy of Medical Sciences, All Rights Reserved. 27/88

(56). Anti-vertigo drug which modulates calcium channels and has prominent labyrinthine suppressant property is
a. Cyproheptadine
b. Cinnarizine
c. Clemastine
d. Cetrizine

Solution. Ans-56: (b) Cinnarizine
Ref: KDT--160
Sol:
Cinnarizine(Stugeron, Stunarone) is an antihistamine which is mainly used for the control of nausea and vomiting due to motion
sickness.
It acts by interfering with the signal transmission between vestibular apparatus of the inner ear and the vomitingcentre of the
hypothalamus.
The disparity of signal processing between inner ear motion receptors and the visual senses is abolished, so that the confusion
of brain whether the individual is moving or standing is reduced.
Vomiting in motion sickness isactually a physiological compensatory mechanism of the brain to keep the individual from moving so
that it can adjust to the signal perception.
[citation needed]
Cinnarizine could be also viewed as a nootropic drug because of its vasorelaxating abilities (due to calcium channel blockage),
which happen mostly in brain.
It is also effectively combined with other nootropics, primarily Piracetam; in such combination each drug potentiate the other in
boosting brain oxygen supply.
Answer. b

(57). SLE-like reactions are caused by all of the following except -
a. Isoniazid
b. Penicillin
c. Hydralazine
d. Griseofulvin

Solution. Ans 57: (b) Penicillin
Ref Read the text below
Sol:
Several broad drug categories have been linked to DILE, including the following:
Antiarrhythmics - Procainamide and quinidine
Antibiotics Minocyclineand isoniazid
Antifungals - Griseofulvin and voriconazole
Anticonvulsants - Valproate, ethosuximide, carbamazepineand hydantoins
Hormonal therapy - Leuprolide acetate
Antihypertensives - Hydralazine, methyldopa, and captopril
Anti-inflammatories - Penicillamine and sulfasalazine
Antipsychotics - Chlorpromazine
Cholesterol-lowering agents - Lovastatin, simvastatin(DISCLE), atorvastatin, and gemfibrozil
Biologics - Interleukins (eg, IL-2), interferons (eg, alfa, beta, gamma), and TNF- (etanercept, infliximab, adalimumab)
Inhalers - Tiotropium bromide inhaler
Other drug categories - Ophthalmic timolol
Answer. b

Copyright 2014 Delhi Academy of Medical Sciences, All Rights Reserved. 28/88
(58). Most specific drug for the treatment of peptic ulcer disease due to chronic use of aspirin is
a. Omeprazole
b. Misoprostol
c. Pirenzipine
d. Ranitidine

Solution. Ans-58: (b) Misoprostol
Ref: KDT-633, 634
Sol:
Misoprostol is approved for use in the prevention of NSAID-induced gastric ulcers.
It acts upon gastric parietal cells, inhibiting the secretion of gastric acid via G-protein coupled receptor-mediated inhibition of
adenylate cyclase, which leads to decreased intracellular cyclic AMP levels and decreased proton pumpactivity at the apicalsurface
of the parietal cell.
Because other classes of drugs, especially H2-receptor antagonists and proton pump inhibitors, are more effective for the
treatment of acute peptic ulcers, Misoprostol is only indicated for use by people who are both taking NSAIDs and are at high risk for
NSAID-induced ulcers, including the elderly and people with ulcer complications. Misoprostol is sometimes co-prescribed with
NSAIDs to prevent their common adverse effect of gastric ulceration(e.g. with Diclofenacin Arthrotec).
Answer. b

(59). Mebendazole may be used for treatment of all of the following conditions except:
a. Enterobiasis
b. Ascariasis
c. Hook worm disease
d. Strongyloidiasis

Solution.
Answer. d

Copyright 2014 Delhi Academy of Medical Sciences, All Rights Reserved. 29/88
(60). The blood culture from a patient of febrile neutropenia has grown pseudomonas aeruginosa,it was found to be a producer of extended
spectrum beta lactamase enzyme. The best choice of antimicrobial therapy should be:
a. Ceftazidime + Amikacin
b. Aztreonam + Amikacin
c. Cefpirome + Amikacin
d. Imipenem + Amikacin

Solution. Ans-60: (d) Imipenem + Amikacin
Ref:Read the text below
Sol:
CARBAPENEMS
Carbapenems are synthetic that differ from the penicillins in that the sulfur atom of the thiazolidine ring has been externalized and
replaced by a carbon atom.
Imipenem is the only drug of this group currently available.
Antibacterial spectrum:
1. Imipenem/cilastatin is the broadest spectrum -lactam antibiotics preparation currently available.
2. Imipenem resists hydrolysis by most -lactam antibiotics
3. The drug plays a role in empiric therapy since it is active against penicillinase-producing grampositive and gram-negative
organisms, anaerobes, an pseudomonas aeruginosa,although other pseudomonas strains are resistant.
Answer. d

(61). A patient with cancer received extreme degree of radiation toxicity. Further history revealed that the dose adjustment of a particular
drug was missed during the course of radiotherapy. Which of the following drugs required a dose adjustment in that patient during
radiotherapy in order to prevent radiation toxicity:
a. Vincristine
b. Dactinomycin
c. Cyclophosphamide
d. 6-Mercaptopurine

Solution. Ans-61: (b) Dactinomycin
Ref:Read the text below
Sol:

Answer. b

Copyright 2014 Delhi Academy of Medical Sciences, All Rights Reserved. 30/88
(62). Gentamycin, obtained from Micromonospora purpura, has become the most commonly used aminoglycoside for acute infections; but is
ineffective against: -
a. Klebsiella
b. Strep.pyogenes
c. Proteus
d. Pseudomonas

Solution. Ans-62: (b) Strep.pyogenes
Ref:Read the text below
Sol:
Gentamicin has a broader spectrum of action: effective against Ps. aeruginosa and most strains of Proteus, E. coli, Klebsiella,
Enterobacter, Serratia.
It is ineffective against M.tuberculosis, Strep. pyogenes and Strep. pneumoniae, but inhibits many Strep.faecalis and some
Staph.aureus.
Answer. b

(63). Which of the following antimicrobials needs dose reduction even in mild renal failure
a. Ciprofloxacin
b. Carbenicillin
c. Cefotaxime
d. Ethambutol

Solution. Ans-63: (d) Ethambutol
Ref: Read the text below
Sol:
Aminoglycosids, Amphotericin B, cephalosporins, vancomycin, flucytosineand ethambutol require dose reduction even in mild
renal failure.
Answer. d

(64). Pneumococcal resistance to penicillin G is mainly acquired by
a. Conjugation
b. Transduction
c. Transformation
d. All of the above

Solution. Ans-64: (c) Transformation
Ref: Read the text below
Sol:
Acquisition of antibiotic resistanceby Transduction is common in Staphylococcal and that of by Transformation in Pneumococcus
and Neisseria.
Vancomycin resistance in enterococci and staphylococcusis mediated by conjugative plasmid.
Answer. c

Copyright 2014 Delhi Academy of Medical Sciences, All Rights Reserved. 31/88
(65). Which of the following is active against atypical mycobacteria
a. Clarithromycin
b. Rifabutin
c. Ciprofloxacin
d. All of the above

Solution. Ans-65: (d) All of the above
Ref: Read the text below
Sol:
Most atypical Mycobacteriaare resistant to the usual antitubercular drugs, though pulmonary disease caused by M. avium complex
or M. Kansasii may respond to prolonged treatment with Rifampicin, Isoniazid and Ethanbutol.
Various combination of drugs that are used are :
Rifabutin
Clofazimine
Quinolones
Newer macrolides like Clarithromycin.
Answer. d

(66). Photo toxicity is caused by all, except
a. Tetracycline
b. Atenolol
c. Quinolones
d. Amiodarone

Solution. Ans-66: (b) Atenolol
Ref: Read the text below
Sol:
A phototoxic substance is a chemical compound which becomes toxic when exposed to light.
Some medicines: tetracycline antibiotics, sulfonamides, amiodarone, quinolones
Many cold pressed citrus essential oilssuch as bergamotoil
Some plant juices: parsleyand hogweed
Others: psoralen
Answer. b

(67). Best for systemic aspergillus infection is
a. Ketoconazole
b. Itraconazole
c. Fluconzole
d. Flucytocine

Solution. Ans-67: (b) Itraconazole
Ref: KDT-722
Sol:
Itraconazole is a newer orallyactive triazole antifungal having a broader spectrum of activity than ketoconazole and fluconazole.
It is preferred over ketoconazole for most systemic mycosis.
Fluconazole is not effective against aspergillosis
Answer. b

Copyright 2014 Delhi Academy of Medical Sciences, All Rights Reserved. 32/88
(68). Which of the following cephalosporins is active against Pseudomonas aeruginosa ?
a. Ceftriaxone
b. Cephalothin
c. Ceftazidime
d. Cefotaxime

Solution. Ans-68: (c) Ceftazidime
Ref: KDT706
Sol:
Ceftazidimeis a third-generation cephalosporin antibiotic. Like other third-generation cephalosporins, it has broad spectrum
activity against Gram-positiveand Gram-negativebacteria.
Unlike most third-generation agents, it is active against Pseudomonas aeruginosa, however it has weaker activity against
Gram-positive microorganisms and is not used for such infections
Ceftazidime is usuallyreserved for the treatment of infections caused by Pseudomonas aeruginosa. It is also used in the empirical
therapy of febrile neutropenia, in combination with other antibiotics.
It is usually given IV or IMevery 812 hours (2 - 3 times a day), with dosage varying by the indication, infection severity,
and/or renal function of the recipient.
Ceftazidine is first line treatment for the rare tropical infection, melioidosis
Answer. c

(69). Which of the following antitubecular drug is contraindicated in pregnancy ?
a. Rifampicin
b. Ethambutol
c. Pyrazinamide
d. Streptomycin

Solution. Ans-69: (d) Streptomycin
Ref: Read the text below.
Sol:
Streptomycincan cause fetal harm when administered to a pregnant woman.
Because streptomycin readily crosses the placental barrier, caution in use of the drug is important to prevent ototoxicity in the
fetus.
If this drug is used during pregnancy, or if the patient becomes pregnant while taking this drug, the patient should be apprised of the
potential hazard to the fetus.
Answer. d

(70). Which of the following is a Rho-kinase inhibitor?
a. Fasudil
b. Nicorandil
c. Amiloride
d. Ranolazine

Solution. Ans 70: (a) Fasudil
Ref Read the text below
Sol:

Fasudil Hydrochloride(INN) is a potent Rho-kinase inhibitor and vasodilator.

Since it was discovered, it has been used for the treatment of cerebral vasospasm, which is often due to subarachnoid hemorrhage,
as well as to improve the cognitive decline seen in stroke victims. It has been found to be effective for the treatment of pulmonary
hypertension.
It was demonstrated in February 2009 that Fasudil could also be used to enhance memoryand improve the prognosis of
Alzheimerspatients
Answer. a

Copyright 2014 Delhi Academy of Medical Sciences, All Rights Reserved. 33/88
(71). Which of the following vasopressors would be preferred in a patient with aortic stenosis?
a. Ephedrine
b. Dopamine
c. Dobutamine
d. Phenylephrine

Solution. Ans 71: (d) Phenylephrine
Ref Read the text below
Sol:
Phenylephrine iscommonly recommended as the most appropriate first-line treatment for hypotension in the patient with aortic
stenosis.
The rationale is 3-fold: (a) if left ventricular (LV) afterload is relatively fixed by the stenotic valve, increasing peripheral vascular
resistance will have less of an effect on myocardial work than it would on an unloaded left ventricle;
(b) increases in diastolic blood pressure will presumably increase CPP, and thus myocardial DO
2
; and (c) reflexive bradycardia
reducesmyocardial consumption of oxygen
Answer. d

(72). Lithium potentiates the action of non-depolarizing muscle relaxants. How many days before administration of the muscle relaxant should
lithium be stopped?
a. 1 day
b. 2 days
c. 3 days
d. 4 days

Solution. Ans 72: (a) 1 day
Ref Read the text below
Sol:
Lithium should be stopped 24 hours before major surgery, but the normal dose can be continued for minor surgery if fluids and
electrolytes are carefully monitored
Answer. a

(73). Orphan drugs are -
a. Drugs for orphans
b. Drugs for treating rare diseases
c. Rare drugs for treating common diseases
d. Drugs that are used very rarely

Solution. Ans 73: (b) Drugs for treating rare diseases
Ref Read the text below
Sol:
An orphan drug is a pharmaceutical agent that has been developed specifically to treat a rare medical condition, the condition
itself being referred to as an orphan disease.
In the US and EU it is easier to gain marketing approval for an orphan drug, and there may be other financial incentives, such as
extended exclusivity periods, all intended to encourage the development of drugs which might otherwise lack a sufficient
profit motive.
The assignment of orphan status to a disease and to any drugs developed to treat it is a matter of public policy in many countries, and
has resulted in medical breakthroughs that may not have otherwise been achieved due to the economics of drug research and
development.
Answer. b

Copyright 2014 Delhi Academy of Medical Sciences, All Rights Reserved. 34/88
(74). Pripaism is seen in poisoning due to -
a. Sea snake
b. Rattle snake
c. Spanish fly
d. Scorpion

Solution. Ans 74: (c) Spanish fly
Ref Read the text below
Sol:
Causes of priapism-
The causative mechanisms are poorly understood but involve complex neurological and vascular factors. Priapism may be associated
with haematologicaldisorders, especially sickle-cell disease, sickle-cell trait, and other conditions such as leukemia,
thalassemia, and Fabry's disease, and neurologic disorderssuch as spinal cordlesions and spinal cord trauma (priapism has
been reported in hanging victims; see death erection).
Priapism may also be associated with glucose-6-phosphate dehydrogenase deficiency, which leads to decreased NADPH levels.
NADPH is a co-factor involved in the formation of nitric oxide, which may result in priapism.
Raised levels of adenosine may also contribute to the condition by causing blood vessels to dilate, thus influencing blood flow into the
penis.
Sickle cell diseaseoften presents special treatment obstacles. Hyperbaric oxygen therapy has also been used with success in some
patients.
Priapism is also found to occur in extreme cases of rabies. Priapism can also be caused by reactions to medications.
The most common medications that causepriapism are intra-cavernous injections for treatment of erectile dysfunction
(papaverine, alprostadil). Other groups reported are antihypertensives, antipsychotics (e.g., chlorpromazine, clozapine), antidepressants
(most notably trazodone), anticoagulants, cantharides (Spanish Fly) and recreational drugs (alcohol, heroin and cocaine). Priapism has
also been linked to achalasia.
[citation needed]
Priapism is also known to occur from bites of the Brazilian wandering spider and the black widow
spider.
[citation needed]
PDE-5 inhibitors have been evaluated as preventive treatment for recurrent priapism.
[7][8]
Answer. c

(75). A young male is brought to the emergency department with nausea, vomiting and breathlessness 2 hours after insecticide ingestion. On
examination, there is pin-point pupils and kerosene-like smell emitting from the patient. Which of the following is not correct regarding
the management of this patient?
a. Activated charcoal has no proven therapeutic role
b. Atropine is the antidote of choice
c. Cholinesterase levels do not have prognostic significance
d. Atropine reverses respiratory muscle weakness

Solution. Ans 75: (d) Atropine reverses respiratory muscle weakness
Ref Read the text below
Sol:
Atropine does not have effect nicotinic receptor
Answer. d

Copyright 2014 Delhi Academy of Medical Sciences, All Rights Reserved. 35/88
(76). Which of the following pairs of drugs is correctly matched with its mechanism of action?
a. Pilocarpine - increased uveoscleral outflow
b. Latanoprost- carbonic anhydrase inhibitor
c. Brimonidine - decreased aqueous formation
d. Betaxolol - increased trabecular outflow

Solution. Ans 76: (c) Brimonidine decreased aqueous formation
Ref Read the text below
Sol:
AGENTS DECREASING SECRETION
Beta blockers-
Timolol
Betaxolol
Carteolol
Levobunolol
Metipranolol

Selective
2
Agonists
Apraclonidine(Lid lag)
Brimonidine(anterior uveitis)

Carbonic anhydrase
inhibitor
Acetazolamide(oral)
Dorzolamide
Brinzolamide
AGENTS ENHANCING OUTFLOW
Trabecular outflow
Cholinomimetics - Pilocarpine (direct acting, short acting) , ecothiophate, demacarium- long acting- ADR- Cataract
Uveoscleral outflow
Prostaglandin F
2
- Latanoprost, bimatoprost (eyelash) and unoprostone
Non selective agonists Epinephrine Dipevefrine
Answer. c

(77). Ocreoplasmin is a recombinant protease that is used for the treatment of -
a. Retinal breaks
b. Diabetic macular edema
c. Vitreo-macular adhesions
d. Sub-macular bleed

Solution. Ans 77: (c) Vitreo-macular adhesions
Ref Read the text below
Sol:
Ocriplasmin(trade name Jetrea) is a recombinant protease with activity against fibronectin and laminin, components of the
vitreoretinal interface.
It is used for treatment of symptomatic vitreomacular adhesion, for which it received FDA approval on 17 October 2012.
It works by dissolving the proteins that link the vitreous to the macula, resulting in posterior detachment of the vitreous from the
retina
Answer. c

Copyright 2014 Delhi Academy of Medical Sciences, All Rights Reserved. 36/88
(78). Which of the following is not true about methanol poisoning?
a. The toxic level of methanol is 1.25 mL/kg
b. Formic acid is responsible for the toxic effects
c. Patient has obscured, snowy vision
d. Fomepizole competitively inhibits aldehyde dehydrogenase enzyme

Solution. Ans 78: (d) Fomepizole competitively inhibits aldehyde dehydrogenase enzyme
Ref Read the text below
Sol:
Median lethal dose istypically 100 mL (4 fl oz) (i.e. 12 mL/kg body weight of pure methanol). Reference dose for methanol is
0,5 mg/kg/day.
Fomepizole competitively inhibits alcohol dehydrogenase
Answer. d

(79). Which of the following intravenous anesthetic agents is contraindicated in epileptic patients posted for general anesthesia?
a. Ketamine
b. Thiopentone
c. Propofol
d. Midazolam

Solution. Ans 79: (a) Ketamine
Ref Read the text below
Sol:
Thiopental in particular is aproven medication in the treatment of status epilepticus. Methohexital can increase ictal activity, and
seizures have been described in patients who received doses sufficient to produce burst suppression of the EEG
Results from studies on theanticonvulsant effects of propofol have been mixed; some data even suggest it has proconvulsant activity
when combined with other drugs. However, propofol has been shown to suppress seizure activity in experimental models and has been
used for the treatment of status epilepticus in humans (Parviainen et al
The effects of ketamine on seizure activity appear mixed, without either strong pro- or anticonvulsant activity. Emergence delirium,
characterized by hallucinations, vivid dreams, and delusions, is a frequent complication of ketamine that can result in serious patient
dissatisfaction and can complicate postoperative management. Delirium symptoms are most frequent in the first hour after emergence
and appear to occur less frequently in children. Benzodiazepines reduce the incidence of emergence delirium
Answer. a

Copyright 2014 Delhi Academy of Medical Sciences, All Rights Reserved. 37/88
(80). A patient posted for surgery has raised intracranial tension. Which of the following anesthetics would be preferred in him?
a. Halothane
b. Isoflurane-
c. Sevoflurane
d. Desflurane-

Solution. Ans 80: (c) Sevoflurane
Ref Read the text below
Sol:
Halothane-This can result in an increase in intracranial pressure, especially in patients with space-occupying intracranial masses,
brain edema, or preexisting intracranial hypertension. ContraindicationsHistory of unexplained jaundice following previous
exposure to halothane.Patients who have developed malignant hyperthermia with prior use.Patients with increased intracranial
pressure.
Isoflurane-There is a modest risk of an increase in intracranial pressure in patients with preexisting intracranial hypertension
Desflurane- desflurane producesan increase in cerebral blood flow and can increase intracranial pressure in patients with poor
intracranial compliance
While sevoflurane can increase intracranial pressure in patients with poor intracranial compliance, the response to hypocapnia is
preserved during sevoflurane anesthesia, and increases in intracranial pressure can be prevented by hyperventilation
All volatile anaesthetics cause a dose dependent increase in cerebral blood flow thorugh direct vasodilation, and thereby
increases intracranial pressure.
Sevoflurane has less influence on CBF and ICP than isoflurane , desoflurane. Halothane has highest influence on CBF and
ICP.
Answer. c

(81). Suxamethonium is available as a clear, colourless liquid. The shelf-life of suxamethonium is
a. 6 months
b. 1 year
c. 2 years
d. 6 years

Solution. Ans 81: (c) 2 years
Ref Read the text below
Sol:
Pharmaceutical particulars

Sodium Acetate B.P
Water for Injections BP

Incompatibilities
Suxamethonium should not be mixed in the same syringe with any other agent especially thiopentone.
Shelf life
Unopened : 18 months
Special precautions for storage

Protect from light.
Store at 2 - 8C.
Do not freeze.


Nature and contents of container

2ml, clear One point cut (OPC) glass ampoules, glass type 1 Ph.Eur. borosilicate glass, packed in cardboard cartons to contain 10 x
2ml ampoules.
Answer. c

Copyright 2014 Delhi Academy of Medical Sciences, All Rights Reserved. 38/88
(82). Which of the following is used as an anti-carving agent in patients of alcohol dependence?
a. Acamprosate
b. Disulfiram
c. Buprenorphine
d. Clonidine

Solution. Ans 82: (a) Acamprosate
Ref Read the text below
Sol:
Anti craving drugs-
Naltrexone
Acamprostate
Serotonin Reuptake Inhibitors
Ondansetron
Topiramate, Baclofen
Rimonabant, a CB1 receptor antagonist
Answer. a

(83). Which fluoroquinolone is highly active against Mycobacterium leprae and is being used in alternative multidrug therapy regimens
a. Norfloxacin
b. Ofloxacin
c. Ciprofloxacin
d. Lomefloxacin

Solution. Ans-83 (b) Ofloxacin
Ref: KDTs 6
th
Ed. Pg. 756
Sol:
Single lesion single dose treatment of leprosy utilizes ROM therapy.
R : rifampicin
O : Ofloxacin
M : Minocycline
Answer. b

(84). The fastest acting schizontocidal drug among the following is
a. Artemether
b. Mefloquine
c. Chloroquine
d. Proguanil

Solution. Ans-84: (a) Artemether
Ref: KDT-792
Sol:
Artemisinin derivativeslike dihydroartemisinin, arteether and artemether etc. are fastest acting antimalarial drugs.
Answer. a

Copyright 2014 Delhi Academy of Medical Sciences, All Rights Reserved. 39/88
(85). All of the following antibiotics act by interfering with cell wall formation except
a. Ceftriaxone
b. Vancomycin
c. Cycloserine
d. Clindamycin

Solution. Ans-85 (d) Clindamycin
Ref: KDTs 6
th
Ed. Pg. 668
Sol:
Drugs inhibiting the cell wall formation are :
Lactams (like penicillins and cephalosporins)
Vanconycin , Cycloserine
Bacitracin , Fosfomycin
Clindamycin acts by inhibiting the protein synthesis
Answer. d

(86). The persistent suppression of bacterial growth that may occur after limited exposure to some antimicrobial drug is called
a. Time dependent killing
b. Post antibiotic effect
c. Concentration dependent killing
d. Sequential blockade

Solution. Ans-86: (b) Post antibiotic effect
Ref: Katzung -- 756
Sol:
Time dependent killing kinetics is shown by aminoglycosides. Here, the killing activity depends upon the length of time, plasma
concentration is above MIC. Concentration dependent killing is shown by lactam drugs.
Here, killing activity depends upon the ratio of plasma concentration to MIC.
Post antibiotic effects is the suppression of bacterial growth after limited exposure to antibiotic.
Answer. b

(87). A folic acid antagonist useful as an anti-neoplastic and immunosuppressant agent is
a. Cyclosporine
b. Azathioprine
c. Doxorubicin
d. Methotrexate

Solution. Ans-87: (d) Methotrexate
Ref: KDT--772
Sol:
Methotrexate is an anti-folateagent that inhibitis DHFR reductase and may result in megaloblastic anemia.
Methotrexate is commonly used (generally in combination with misoprostol) to terminate pregnancies during the early stages (i.e., as
an abortifacient). It is also used to treat ectopic pregnancies
Answer. d

Copyright 2014 Delhi Academy of Medical Sciences, All Rights Reserved. 40/88
(88). Tamoxifen is used to treat some breast cancers because of its ability to
a. Utilize its androgenic properties in retarding tumor growth
b. Prevent estrogen synthesis by the ovary
c. Enhance glucocorticoid treatment
d. Act as an estrogen antagonist

Solution. Ans-88: (d) Act as an estrogen antagonist
Ref: Read the text below.
Sol:
Tamoxifen is an estrogen antagonist used in the treatment of breast cancer.
Postmenopausal women with metastases to soft tissue and whose tumors contain an estrogen receptor are more likely to respond
to this agent.
Little benefit is derived from tamoxifen if the tumor does not have estrogen receptors.
Answer. d

(89). An effective antidiarrheal agent that inhibits peristaltic movement is
a. Clonidine
b. Bismuth subsalicylate
c. Oral electrolyte solution
d. Diphenoxylate

Solution. Ans-89: (d) Diphenoxylate
Ref: Read the text below.
Sol:
Diphenoxylate is a piperidine opioidthat is related to meperidine. It inhibits peristalsis and, hence,increases the passage time of
the intestinal bolus.
It is combined with atropine to discourage use as a street drug.
Atropine has little effect on peristalsis.Clonidine, bismuth subsalicylate, and rehydration therapy are all useful in some types of
diarrhea, but none of them inhibit peristalsis.
Answer. d

(90). The steatorrhea of pancreatic insufficiency can best be treated by
a. Misoprostol
b. Bile salts
c. Pancrelipase
d. Secretin

Solution. Ans-90: (c) Pancrelipase
Ref: Read the text below.
Sol:
Pancrelipase is an alcoholicextract of hog pancreas that contains lipase, trypsin, and amylase.
It is effective in reducing the steatorrhea of pancreatic insufficiency.
None of the other drugs mentioned have significant action in the digestion of fats.
Answer. c

Copyright 2014 Delhi Academy of Medical Sciences, All Rights Reserved. 41/88
(91). A reduction in insulin release from the pancreas may be caused by which of the following diuretics?
a. Triamterene
b. Chlorothiazide
c. Spironolactone
d. Acetazolamide

Solution. Ans-91: (b) Chlorothiazide
Ref: Read the text below.
Sol:
An adverse reaction that is reported to occur occasionally with the thiazides, such as chlorothiazide, is hyperglycemia.
In addition, hyperglycemia may occur with thiazide-related compounds (chlorthalidone and metolazone) and the high-ceiling
diuretics (ethacrynic acid, furosemide, and bumetanide).
The proposed mechanism for the elevation in blood glucose appears to be related to a decrease in insulin release from the
pancreas.
In addition, increased glycogenolysis,decreased glycogenesis, and a reduction in the conversion of proinsulin to insulin may also
be involved in the hyperglycemic response.
Diazoxide, a nondiuretic thiazide, is given to treat hypoglycemia in certain conditions.However, diazoxide is used more often to control
hypertensive emergencies
Answer. b

(92). Which of the following agents is a selective dopamine receptor (D2) agonist?
a. Fluphenazine
b. Bromocriptine
c. Promethazine
d. Haloperidol

Solution. Ans-92: (b) Bromocriptine
Ref: Read the text below.
Sol:
Central dopamine receptorsare divided into D1 and D2 receptors. Antipsychotic activity is better correlated to blockade of D2
receptors.
Haloperidol, a potent antipsychotic, selectively antagonizes at D2 receptors. Phenothiazine derivatives, such as chlorpromazine,
fluphenazine, and promethazine, are not selective for D2 receptors.
Bromocriptine, a selective D2 agonist, is useful in the treatment of parkinsonism and hyperprolactinemia.
It produces fewer adverse reactions than do nonselective dopaminereceptor agonists.
Answer. b

(93). In comparing the following neuroleptics, which is most likely associated with constipation, urinary retention, blurred vision, and dry
mouth?
a. Chlorpromazine
b. Clozapine
c. Olanzapine
d. Sertindole

Solution. Ans-93: (a) Chlorpromazine
Ref: Read the text below.
Sol:
The phenothiazines as a class are the most potent anticholinergics of the neuroleptics. Tolerance to their anticholinergic effects
occurs in most patients.
Cholinomimetic agentsmay be used to overcome symptoms that persist.
Answer. a

Copyright 2014 Delhi Academy of Medical Sciences, All Rights Reserved. 42/88
(94). A patient exhibiting multiple facial tics, aggressive outbursts of behavior, and spontaneous repetitive foul language is best treated with
which of the following agents?
a. Levodopa
b. Clozapine
c. Thioridazine
d. Haloperidol

Solution. Ans-94: (d) Haloperidol
Ref: Read the text below.
Sol:
Tourettes syndromeis effectively treated with haloperidol, a high-potency antipsychotic.
If patients are unresponsive or do not tolerate haloperidol, they might be switched to pimozide.
Answer. d

(95). Which of the following is an antidepressant agent that selectively inhibits serotonin (5-HT) uptake with minimal effect on norepinephrine
uptake?
a. Protriptyline
b. Maprotiline
c. Fluoxetine
d. Desipramine

Solution. Ans-95: (c) Fluoxetine
Ref: Read the text below.
Sol:
The tricyclics and secondgeneration antidepressants act by blocking serotonin or norepinephrine uptake into the presynaptic
terminal.
Fluoxetine selectively inhibits serotonin uptake with minimal effects on norepinephrine uptake.
Protriptyline, maprotiline, desipramine, and amoxapine have greater effect on norepinephrine uptake.
Answer. c

(96). Carbidopa is useful in the treatment of Parkinson's disease because it
a. Is a precursor of levodopa
b. Is a dopaminergic receptor agonist
c. Prevents peripheral biotransformation of L-dopa
d. Prevents a breakdown of dopamine

Solution. Ans-96: (c) Prevents peripheral biotransformation of L-dopa
Ref: Read the text below.
Sol:
Carbidopa is an inhibitor of aromatic L-amino acid decarboxylase.
It cannot readily penetrate the central nervous system (CNS) and, thus, decreases the decarboxylation of L-dopa in the peripheral
tissues.
This promotes an increased concentration of L-dopa in the nigrostriatum, where it is converted to dopamine.
In addition, the effective dose of L-dopa can be reduced.
Answer. c

Copyright 2014 Delhi Academy of Medical Sciences, All Rights Reserved. 43/88
(97). Successful treatment with single drug in Hodgkin's disease has been seen with:
a. Procarbazine
b. Mithramycin
c. Mechlorethamine
d. Radiophosphorus

Solution. Ans-97: (c) Mechlorethamine
Ref:Read the text below
Sol:
Mechlorethamine is used primarily in the treatment of Hodgkins disease as part of the MOPP regimen (Mechlorethamine,
Oncovin, Prednisone, Procarbazine)
Is also useful in the treatment of some solid tumors.
Answer. c

(98). Of the patients who are allergic to penicillin, what percentage will have an allergic reaction when given cephalosporin?
a. Fewer that 1%
b. 5 to 15 %
c. 2 to 5 %
d. 1 to 2 %

Solution. Ans-98: (b) 5 to 15 %
Ref:Read the text below
Sol:
The cephalosporins should be avoided or used with caution in individuals allergic to penicillins (about 5 to 15% show cross-
sensitivity).
In contrast, the incidence of allergic reactions to cephalosporins is 1-2% in patients without a history of allergy to
penicillins.
Answer. b

Copyright 2014 Delhi Academy of Medical Sciences, All Rights Reserved. 44/88
(99). A 60 years old female patient presents with microbial infection,decision is made to treat her with antibiotics as an outpatient. Routine
biochemistry tests reveals creatinine clearance of 28/ml/ min.Drug least likely to require a dosage adjustment is:
a. Acyclovir
b. Cephalexin
c. Metronidazole
d. Vancomycin

Solution. Ans-99: (c) Metronidazole
Ref:Read the text below
Sol:
ANTIMICROBIAL NEEDING DOSE REDUCTION IN RENAL FAILURE
Answer. c

(100). An immunocompromised patient who is being treated with drugs such as didanosine, efavirenz, Nevirapine, and Ritonavir, develops
pancreatitis and peripheral neuropathy. If there changes are related to his drug treatment; the most likely cause is: -
a. Didanosine
b. Efavirenz
c. Nevirapine
d. Ritonavir

Solution. Ans-100: (a) Didanosine
Ref:Read the text below
Sol:
Answer. a

Copyright 2014 Delhi Academy of Medical Sciences, All Rights Reserved. 45/88
(101). Diuretic agent having therapeutic value in the treatment of acute pulmonary oedema:
a. Spironolactone
b. Loop diuretics
c. Triamterene
d. Amiloride.

Solution. Ans-101: (b) Loop diuretics
Ref:Read the text below
Sol:
Answer. b

(102). Antimicrobial agents are used primarily in the treatment of:
a. Uric acid stones
b. Struvite stones
c. Cystine stones
d. Hypocitraturia

Solution. Ans-102: (b) Struvite stones
Ref:Read the text below
Sol:
STRUVITE STONES
These stones are a result of urinary infection with bacteria, usually Proteus species, which possess urease, an enzyme that
degrades urea to NH3 and CO2.
Chronic Proteus infection can occur because of impaired urinary drainage, urologic instrumentation or surgery.
Complete removal of the stone with subsequent sterilization of the urinary tract is the treatment of choice for patients who can
tolerate the procedures.
Antimicrobial treatment is best reserved for dealing with acute infection and maintenance of a sterile urine.
Answer. b

Copyright 2014 Delhi Academy of Medical Sciences, All Rights Reserved. 46/88
(103). Vectibix (panitumumab), a recombinant, human IgG2 kappa monoclonal antibody used in various tumors acts by :
a. Inhibiting multiple receptor tyrosine kinases
b. Binding specifically to the human Epidermal Growth Factor Receptor (EGFR)
c. Binds to the receptor binding site of active forms of VEGF-A
d. The induction of phospholipase A2 inhibitory

Solution. Ans-103: (b) Binding specifically to the human Epidermal Growth Factor Receptor (EGFR)
Ref:Read the text below
Sol:
Proteins
VECTIBIX (PANITUMUMAB)
For the treatment of colorectal cancer; Amgen; Approved September 2006
Vectibix is a recombinant, human IgG2 kappa monoclonal antibody that binds specifically to the human Epidermal Growth
Factor Receptor (EGFR).
Overexpression of EGFR is detected in many human cancers, including those of the colon and rectum..
Vectibix is specifically indicated for the treatment of EGFR-expressing, metastatic colorectal carcinoma with disease progression
on or following fluoropyrimidine-, oxaliplatin-, and irinotecan- containing chemotherapy regimens.
Answer. b

(104). Resistance to zidovudine develops due to :-
a. Mutation at reverse transcriptase
b. Increased efflux of the drug from inside the cell
c. Increased metabolism of the drug
d. Decreased zidovudine 5 triphosphate Formation

Solution. Ans-104: (a) Mutation at reverse transcriptase
Ref:Read the text below
Sol:
"Resistance to AZT (Zidovudine) occurs by point mutations which alter the reverse transcriptase enzyme."
Answer. a

(105). 26 years old pregnant female presented in medicine OPD with difficulty in breathing, pulmonary function tests confirms the diagnosis of
asthma. Which drug should be limited to third line therapy because of difficulties in its administration and potential for serious adverse
effects : -
a. Theophylline
b. Inhaled cromolyglycate
c. Inhaled b2 adrenergic agonist
d. Inhaled glucocorticoids

Solution. Ans-105: (a) Theophylline
Ref:Read the text below
Sol:
ASTHMA DURING PREGNANCY
Maternal and fetal adverse effect are rare when inhaled b2 adrenergic agonists are used at the recommended doses. Systemic b2
adrenergic agonists can cause fetal tachycardia and neonatal tachycardia, hypoglycemia and tremor.
Inhaled b2 adrenergic agonists appear to be more effective and do not carry the risk of uterine vasoconstriction.
Inhaled cromoglycate is considered particularly safe in pregnancy, because it is extremely poorly absorbed from the gastrointestinal
tract.
Inhaled glucocorticoids also are considered relatively safe in pregnancy.
Theophylline is now infrequently used, in part of because of its limited effectiveness and narrow therapeutic window.
In practice, theophylline should be limited to third line therapy after inhaled anti-inflammatory agents and b2adrenergic agonists,
because of the above difficulties in its administration and the potential for serious adverse effects
Answer. a

Copyright 2014 Delhi Academy of Medical Sciences, All Rights Reserved. 47/88
(106). Mainstay of therapy for alveolitis present in interstitial lung disease is: -
a. Cyclophosphamide
b. Prednisone
c. Azathioprine
d. Colchicine

Solution. Ans-106: (b) Prednisone
Ref:Read the text below
Sol:
Glucocorticoids are the mainstay of therapy for suppression of the alveolitis present in ILD, but the success rate is low.
Glucocorticoid therapy is recommended for symptomatic ILD patients with idiopathic interstitial pneumonias, eosinophilic
pneumonias and sarcoidosis
In organic dust disease, glucocorticoids are recommended for both the acute and chronic stages.
Answer. b

(107). Nesiritide is a recombinant form of :-
a. ANP
b. BNP
c. TNF-a
d. IL-1

Solution. Ans-107: (b) BNP
Ref:Read the text below
Sol:
Nesiritide, a form of human brain natriuretic peptide (BNP) has been approved for treatment of dyspnoea due to congestive failure.
BNP IS SECRETED BY VENTRICULAR CARDIAC MYOCYTES IN RESPONSE TO STRETCH.
In setting of heart failure, effect of BNP counteract the effects of angiotensin & norepinephrine by producing vasodilation, natriuresis
& diuresis.
Nesiritide lowers right & left side cardiac filling pressure without direct chronotropic & ionotropic effect.
Primary side effect is hypotension.
Answer. b

(108). To achieve pleurodesmosis in malignant pleural effusion, the drug used is:
a. Polymyxin B
b. Chloromycetin
c. 1% Betadine
d. Doxycycline

Solution. Ans-108: (d) Doxycycline
Ref:Read the text below
Sol:
Patient with a malignant pleural effusion are treated symptomatically for the most part, since the presence of the effusion
indicate disseminated disease and most malignancies associated with pleural effusion are not curable with chemotherapy.
The only symptom that can be attributed to the effusion itself is dyspnea.
If the patients lifestyle is compromised by dyspnea, and if the dyspnea is relieved with a therapeutic thoracentesis, then one of the
following procedure should be considered:
Tube thoracostomy with the instillation of a sclerosing agent such as doxycycline, 500 mg
Outpatient insertion of a small indwelling catheter.
Answer. d

Copyright 2014 Delhi Academy of Medical Sciences, All Rights Reserved. 48/88
(109). Mechanism of action of rifampicin is by :-
a. Inhibition of RNA dependent DNA polymerase
b. Inhibition of DNA dependent RNA polymerases
c. Interferes with translocation.
d. Inhibitits synthesis of mycolic acid, that form part of mycobacterial cell wall.

Solution. Ans-109: (b) Inhibition of DNA dependent RNA polymerases
Ref:Read the text below
Sol:
Rifampin inhibits DNA dependent RNA polymerase of mycobacteria & other microorganisms by forming a stable drug-enzyme
complex leading to suppression of initiation of chain formation (but not chain elongation) in RNA synthesis.
Inhibition of synthesis of mycolic acid is mechanism of isoniazid.
Answer. b

(110). A diabetic female on INH and rifampin for TB suffers DVT: She is started on warfarin: PT is not raised; next step should be:
a. Long term heparin therapy
b. Replace warfarin with acecoumarin
c. Switch ethambutol for rifampin
d. Use LMW heparin.

Solution. Ans-110: (c) Switch ethambutol for rifampin
Ref:Read the text below
Sol:
Rifampin is a microsomal enzyme inducer enhances its own metabolism as well as that of many drugs including warfarin, oral
contraceptives, corticosteroids sulfonylureas, digitoxin, HIV protease inhibitors, ketoconazole etc.
Answer. c

(111). Mirtazapine is useful in :
a. Depression
b. Insomnia
c. Schizophrenia
d. Epilepsy

Solution. Ans-111: (a) Depression
Ref: Read the text below.
Sol:
Mirtazapine is a recently introduced antidepressant. It has a novel mechanism of action of relasing NA and serotonin by blocking
their hetero-receptors and auto-receptors.
Mirtazapine also has antihistaminic and sedative properties, but is not an anticholinergic.
Answer. a

Copyright 2014 Delhi Academy of Medical Sciences, All Rights Reserved. 49/88
(112). Cheese reaction occurs due to which of the following in the food ?
a. Tyramine
b. Serotonin
c. Norepinephrine
d. Epinephrine

Solution. Ans-112: (a) Tyramine
Ref: Read the text below.
Sol:
Cheese contains a high quantity of tyramine, which act basically as an indirect sympathomimetic.
MAO-inhibitors inhibit degradation of tyramineand so the accumulated large amount of tyramine causes hypertensive crisis.
Other foods containing high quantities of tyramine also causes cheese reaction with MAO-inhibitors e.g. beer, wines, pickled meat
and fish and yeast extract.
Answer. a

(113). Which of the following is the most sensitive test for the diagnosis of herpes simplex (HSV) meningitis in a newborn infant?
a. HSV IgG antibody
b. HSV polymerase chain reaction (PCR)
c. HSV culture
d. Cerebrospinal fluid (CSF) protein analysis

Solution. Ans 113: (b) HSV polymerase chain reaction (PCR)
Reference Read the text below
Sol:
HSV meningitisor encephalitis is difficult to diagnose by laboratory tests as there is a low titer of virus present in the CSF.
Neonatal HSV infectsthe child during the birth process. While culture, Tzanck smear, and even antibody tests may be useful in
adults, particularly those with HSV-rich lesions, they are not useful for CSF testing.
Only PCR is sensitive enough to detect HSV DNA in the CSF. Once diagnosed rapidly, HSV encephalitis or meningitis can
be treated with acyclovir.
Answer. b

(114). Zopiclone:
a. Is an ultra short-acting benzodiazepine
b. Is the hypnotic of choice in a breast-feeding mother
c. Is a more effective anticonvulsant than clonazepam
d. Is associated with drug dependence

Solution. Ans 114: (d) Is associated with drug dependence
Reference Read the text below
Sol:
Zopicloneis a non-benzodiazepine hypnotic which enhances GABA activity
Answer. d

Copyright 2014 Delhi Academy of Medical Sciences, All Rights Reserved. 50/88
(115). The treatment of choice for " Familial Mediterranean fever" is
a. Cyclophosphamide
b. Cyclosporine
c. Colchicine
d. Quinine

Solution. Ans 115: (c) Colchicine
Reference H
16
-1794
Sol.
The treatment of choice for Familial Mediterranean fever (FMF) is daily oral co1chicine,which decreases the frequency and
intensity of attacks and prevent the development of amyloidosis in compliant patients.
Intermittent dosingat the onset of attacks is not as effective as daily prophylaxis and is of unproven value in preventing
amyloidosis.
The usual adult dose of colchicine is 1.2 to 1.8 mg/d, which causes substantial reduction in symptoms in two-thirds of patients
Common side effects of colchicine include bloating, abdominal IPS, lactose intolerance, and diarrhea. They can be minimized by
starting at a low dose and gradually advancing as tolerated, splitting the dose, use of simethicone for flatulence, and avoidance of
dairy products .
lf taken by either parent at the time of conception, colchicine may cause a small increase in the risk of trisomy 21 (Down
syndrome)
Answer. c

(116). Which one of the following antibiotics inhibits dihydrofolate reductase?
a. Amdinocillin
b. Amphotericin
c. Chloramphenicol
d. Trimethoprim

Solution. Ans 116: (d) Trimethoprim
Reference Read the text below
Sol:
Trimethoprim (TMP), a diaminopyrimidine, is a folic acid antagonist.
Although TMPis commonly used in combination with sulfa drugs, its mode of action is distinct.
TMP is structurally similar to the pteridine portion of dihydrofolate and prevents the conversion of folic acid to
tetrahydrofolic acid by inhibition of dihydrofolate reductase.
Fortunately, this enzyme in humans is relatively insensitive to TMP.
Answer. d

(117). A drug is more likely to cause toxicity in elderly patients due to all of the following reasons except:
a. Decreased renal excretion of drugs.
b. Decreased volume of distribution
c. Decreased hepatic metabolism
d. Increased receptor sensitivity

Solution. Ans 117: (b) Decreased volume of distribution
Reference H
16
-39
Sol:
CHANGES SEEN IN ELDERLY
Decreasedlean body mass
DecreasedTotal body water
Decreasedrenal excretion of drugs.
Decreasedhepatic metabolism of drug.
Increased receptor sensitivity in target organs.
Increased volume of distribution for fat soluble drugs
Answer. b
Copyright 2014 Delhi Academy of Medical Sciences, All Rights Reserved. 51/88

(118). Mainserin:
a. Inhibits MAO
b. Is a powerful anticholinergic agent
c. Is less sedative than imipramine
d. Causes orthostatic hypotension

Solution. Ans 118: (d) Causes orthostatic hypotension
Reference Read the text below
Sol:
Mainserin is a sedative antidepressant which has little anticholinergic and cardiac toxicity( which may help reduce death from
over-dose) but is associated with orthostatic hypotension and blood dyscrasias.
Answer. d

(119). Among anti-tumor drugs, specific adverse effect of busulfan is:
a. Hyperuricaemia
b. Pulmonary fibrosis
c. Sterility
d. Renal Damage

Solution. Ans 119: (b) Pulmonary fibrosis
Reference KDT
5
-772 GG
10
-1398.
Sol:
BUSULFAN
Alkylating agent.
Highly specific for myeloid elements.
Most sensitive Granulocyte precusors.
2
nd
most sensitive - platelets
3
rd
most sensitive RBC
Drug of choice chronic myeloid leukemia
ADR
Pulmonary fibrosis (specific adverse effect)
Hyperuricaemia (common adverse effect)
Thrombocytopenia
Occasional instances of nausea, vomiting, diarrhoea, impotence, sterility, amenorrhea, and fetal malformation.
Answer. b

(120). Which of the following is particularly less sensitive to the pharmacological actions of morphine:
a. Young children
b. Patients with hepatic failure
c. Patients with renal failure
d. Patients with hyperthyroidism

Solution. Ans 120: (d) Patients with hyperthyroidism
Reference Read the text below
Sol:
Patients with hyperthyroidismare particularly less sensitive to the pharmacological actions of morphine
Answer. d

Copyright 2014 Delhi Academy of Medical Sciences, All Rights Reserved. 52/88
(121). For the treatment of a patient with Legionella pneumophila, which of the following is the drug of choice?
a. Penicillin G
b. Chloramphenicol
c. Erythromycin
d. Streptomycin

Solution. Ans 121: (c) Erythromycin
Reference Read the text below
Sol:
Erythromycin, a macrolide antibiotic, was initially designed to be used in penicillin-sensitive patients with streptococcal or
pneumococcal infections.
Erythromycin has become the drug of choice for the treatment of pneumonia caused by Mycoplasma and Legionella.
Answer. c

(122). Pneumococcus producing mucoid colonies most often is type:
a. I
b. II
c. III
d. IV

Solution. Ans 122: (c) III
Reference Read the text below
Sol:
Pneumococci are identified in the clinical laboratory as catalase-negative, gram-positive cocci that grow in pairs or chains
and cause -hemolysis on blood agar.
More than 98% of pneumococcal isolates are susceptible to ethylhydrocupreine (optochin), and virtually all pneumococcal colonies
are dissolved by bile salts.
Peptidoglycan and teichoic acid are the principal constituents of the pneumococcal cell wall, whose integrity depends on the
presence of numerous peptide side chains cross-linked by the activity of enzymes such as trans- and carboxypeptidases.
Some strains that develop abundant capsular material (type 3 and 7) from large mucoid colonies.
Answer. c

(123). Which of the following is the antibiotic of choice for Lyme disease?
a. Ampicillin
b. Erythromycin
c. Vancomycin
d. Ceftriaxone

Solution. Ans 123: (d) Ceftriaxone
Reference Read the text below
Sol:
Lyme disease, caused by B. burgdorferi, has been treated with penicillin, erythromycin, and tetracycline.
Treatment failures have been observed. Ceftriaxone has become the drug of choice, particularly in the advanced stages of Lyme
disease
Answer. d

Copyright 2014 Delhi Academy of Medical Sciences, All Rights Reserved. 53/88
(124). Which one of the following antibiotics binds to penicillin-binding protein-2 (PBP-2)
a. Penicillin
b. Amdinocillin
c. Amphotericin
d. Chloramphenicol

Solution. Ans 124: (b) Amdinocillin
Reference Read the text below
Sol:
The antibiotics in these questions have significantly different modes of action.
Recent evidence suggests that while penicillin inhibits the final cross-linking of the cell wall, it also binds to penicillin-binding
proteins and inhibits certain key enzymes involved in cell-wall synthesis.
The mechanism is complex. Amdinocillin, although classified as a penicillin, selectively binds to penicillin-binding protein-2
(PBP-2).
Binding to PBP-2 results in aberrant cell-wall elongation and spherical forms, seen when E. coli, for example, is exposed to
mecillinam.
Answer. b

(125). A young female with bipolar disorder who failed to respond to adequate lithium therapy should be prescribed
a. Chlorpromazine
b. Amitriptyline
c. Carbamazepine
d. Haloperidol

Solution. Ans-125: (c) Carbamazepine
Ref: Read the text below.
Sol:
About half of the patients of bipolar disorders show inadequate response to lithium therapy or do not tolerate lithium well.
Carbamazepine or sodium Valproate is a good alternative for such patients.
Answer. c

(126). False statement regarding Lithium is:
a. Maximum plasma concentration is avoided due to low therapeutic index.
b. Contraindicated in pregnancy.
c. No individual variation in the rate of excretion.
d. 80% Reabsorbed in the proximal convoluted tubule

Solution. Ans-126: (c) No individual variation in the rate of excretion.
Ref: Read the text below.
Sol:
There is marked individual variation in the rate of lithium exceretion. Thus, with the same daily dose, different individuals attain
much different plasma concentration.
About 1 mEq/L is the therapeutic serum level of Li and toxicity occurs even with serum levels little above 1.5 mEq/L. So, even
therapeutic maximum plasma concentrations are avoided.
Li is contraindicated during pregnancy; fetal goiter and other congenital abnormalities and occur.
Li is excreted by kidneys in much the same way as Na
+
. About 80% filtered Li is reabsorbed in the proximal convoluted tubule.
However, when Na
+
is restricted, a larger fraction of filtered Na
+
and Li
+
is reabsorbed.
Answer. c

Copyright 2014 Delhi Academy of Medical Sciences, All Rights Reserved. 54/88
(127). Which of the following drugs acts via the T type calcium channel antagonism:
a. Ethosuxccimide
b. Phenytoin
c. Gabapentin
d. Lamotrigine

Solution. Ans-127: (a) Ethosuxccimide
Ref: Read the text below.
Sol:
Ethosuccimide is unique in its mechanismof action that it blocks T-type calcium channels and is valuable in Petitmal epilepsy for
which it is the drug of choice.
Ethosuccimide has been used to treat convulsions.Three cases of lupus-like syndrome without renal involvementhave been reported
involving this drug
Answer. a

(128). A drug that specifically enhances metabolically the activity of brain dopamine is
a. Benztropine
b. Selegiline
c. Trihexyphenidyl
d. Bromocriptine

Solution. Ans-128: (b) Selegiline
Ref: Read the text below.
Sol:
Selegiline inhibits MAO-B,thus delaying the metabolic breakdown of dopamine. It is effective alone in parkinsonism and increases
the effectiveness of L-dopa.
Benztropine and trihexyphenidyl are cholinergic antagonists in the brain; bromocriptine is a dopamine receptor agonist.
Chlorpromazine is an antipsychotic drug with antiadrenergic properties.
Answer. b

(129). Of the following antiepileptic agents, which is associated with causing psychosis?
a. Phenobarbital
b. Ethosuximide
c. Phenytoin
d. Vigabatrin

Solution. Ans-129: (d) Vigabatrin
Ref: Read the text below.
Sol:
Vigabatrincan induce psychosis.
It is recommended that it not be used in patients with preexisting depression and psychosis.
Answer. d

Copyright 2014 Delhi Academy of Medical Sciences, All Rights Reserved. 55/88
(130). The skeletal muscle relaxant that acts directly on the contractile mechanism of the muscle fibers is
a. Gallamine
b. Baclofen
c. Pancuronium
d. Dantrolene

Solution. Ans-130: (d) Dantrolene
Ref: Read the text below.
Sol:
There are three major classes of skeletal muscle relaxants: (1) peripherally acting, (2) centrally acting, and (3) direct-acting.
The peripherally acting drugs include the nondepolarizing (e.g., tubocurarine, gallamine, pancuronium) and depolarizing (e.g.,
succinylcholine, decamethonium) neuromuscular blockers that antagonize ACh at the muscle endplate (i.e., at NM receptors).
Centrally acting skeletal muscle relaxants (e.g., diazepam, cyclobenzaprine, baclofen) interfere with transmission along the
monosynaptic and polysynaptic neural pathways in the spinal cord.
Dantrolene, the only directacting skeletal muscle relaxant, affects the excitation-contraction coupling mechanism of skeletal
muscle by depressing the release of ionic Ca from the sarcoplasmic reticulum to the myoplasma.
The drug is also useful in the prevention and management of malignant hyperthermia induced by generalanesthetics.
Answer. d

(131). The most dangerous adverse reaction to the administration of methimazole is
a. Hypothyroidism
b. Arthralgia
c. Jaundice
d. Agranulocytosis

Solution. Ans-131: (d) Agranulocytosis
Ref: Read the text below.
Sol:
Methimazole is classified as a thioamideand is used in the treatment of hyperthyroidism. It prevents the organification of I by
blocking the oxidation of I to active I and also inhibits coupling of iodotyrosines.
Excessive treatment with this drug may induce hypothyroidism. Some other adverse reactions reported for methimazole include
skin rash, fever, jaundice, nephritis, arthralgia, and edema.
Agranulocytosis, which is a very serious reaction and may be fatal, is the most dangerous adverse reaction, but it occurs in less than
1% of patients.
Patients should be carefully monitored while they are taking this medication because agranulocytosis appears without warning.
Answer. d

(132). Which of the following enzymes is inhibited by drugs of the class to which amoxicillin belongs?
a. Alanine racemase
b. Beta lactamase
c. Transglycosylase
d. Transpeptidase

Solution. Ans 132: (d) Transpeptidase
Reference Read the text below
Sol:
All of the beta-lactam antibiotics (penicillins,cephalosporins, carbapenems, monobactams) inhibit one of the final steps in
the synthesis of the bacterial cell wall, cross-linking of peptidoglycan chains by transpeptidase.
Alanine racemaseis inhibited by cycloserine, which is used in the treatment of M. tuberculosis.
Beta lactamaseis inhibited by drugs (e.g., clavulanic acid, sulbactam, tazobactam) used to prevent resistance by bacteria that possess
it.
Enolpyruvate transferase is inhibited by fosfomycin, which is used for urinary tract infections.
Transglycosylaseanother enzyme important in cell wall cross linking, is inhibited by vancomycin.
Answer. d

Copyright 2014 Delhi Academy of Medical Sciences, All Rights Reserved. 56/88
(133). Which of the following drugs is selectively antifungal because of the difference between cholesterol (in mammalian cell membranes) and
ergosterol (in fungal membranes)?
a. Amphotericin B
b. Caspofungin
c. Fluconazole
d. Griseofulvin

Solution. Ans 133: (a) Amphotericin B
Reference Read the text below
Sol:
Amphotericin Bbinds selectively to ergosterol and forms artificial pores in fungal membranes.These pores result in loss of
homeostasis and death of the fungal cell.
Caspofungin is a member of the echinocandins,the newest antifungal antibiotics. These agents disrupt the fungal cell wall by
inhibiting the synthesis of beta(1-3) glucan.
Fluconazole and other azoles(ketoconazole, itraconazole,voriconazole, posaconazole) inhibit ergosterol synthesis. Griseofulvin
interferes with microtubule function and cell division in fungi.
Terbinafine inhibits squalene epoxidase in fungi and reduces ergosterol synthesis.
Answer. a

Copyright 2014 Delhi Academy of Medical Sciences, All Rights Reserved. 57/88
(134). Medications contraindicated during breast feeding include all except.
a. Bromocriptine
b. Cyclosporine
c. Methotrexate
d. Prednisolone

Solution. Ans 134: (c) Methotrexate
Reference Read the text below
Sol:
Answer. c

Copyright 2014 Delhi Academy of Medical Sciences, All Rights Reserved. 58/88
(135). Which one of the following drugs has narrow therapeutic range?
a. Propanolol
b. Digoxin
c. Piroxicam
d. Prazosin

Solution. Ans 135: (b) Digoxin
Reference Read the text below
Sol:
The therapeutic index (also known as therapeutic ratio), is a comparison of the amount of a therapeutic agent that causes the
therapeutic effect to the amount that causes death. Quantitatively, it is the ratio given by the lethal dose divided by the therapeutic dose.
A therapeutic index is the lethal dose of a drug for 50% of the population (LD
50
) divided by the minimum effective dose for 50% of the
population (ED
50
).
Generally, a drug or other therapeutic agentwith a narrow therapeutic range (i.e. with little difference between lethal and
therapeutic doses) may have its dosage adjusted according to measurements of the actual blood levels achieved in the person taking it.
This may be achieved throughtherapeutic drug monitoring (TDM) protocols. The therapeutic index for diazepam is somewhat
forgiving, about = 100. Other drugs, however are much less so, such as Digoxin, which has an index of 2 or 3.
Other examples of drugs with a narrow therapeutic range, that may require drug monitoring both to achieve therapeutic levels
and minimize toxicity, include: dimercaprol, theophylline, warfarin and lithium carbonate.
Some antibiotics require monitoring to balance efficacy with minimizing adverse effects,
including: gentamicin, vancomycin, amphotericin B, and polymyxin B.
Answer. b

(136). Apatient has been treated for bipolar disorder for several months. Which of the following is a common adverse effect of lithium
carbonate?
a. Dry mouth
b. Hyperthyroidism
c. Leukopenia
d. Nephrogenic diabetes insipidus

Solution. Ans 136: (d) Nephrogenic diabetes insipidus
Reference Read the text below
Sol:
Nephrogenic diabetes insipidusis a common adverse effect of lithium at therapeutic dosage.
Dry mouth and other anticholinergic toxicitiesare common with tricyclic antidepressants and some phenothiazines.
Lithium may induce hypothyroidism, but not hyperthyroidism .
Similarly, lithium increases rather than decreases white blood cell count.
Finally, lithium often causes tremor, but not parkinsonism .
Answer. d

(137). Methadone has been used in opioid addiction maintenance clinics with partial success. Its ability to reduce the use of heroin and
morphine by recovering addicts can be ascribed to which of the following mechanisms?
a. Antidepressant effects at serotonin (5-HT) receptors
b. Antagonist actions at the mu opioid receptor
c. Blockade of the metabolic activation of heroin and morphine to their active forms
d. Long-lasting saturation of the mu opioid receptor and development of tolera

Solution. Ans 137: (d) Long-lasting saturation of the mu opioid receptor and development of tolera
Reference Read the text below
Sol:
Methadone is a typical strong agonist at opioid mu receptorsand differs from morphine chiefly in its long duration of binding to
the receptor and long elimination half-life.
It therefore produces a prolonged opioid effect that causes tolerance to the euphoric action of short-acting heroin and
morphine.
Methadone has no opioid antagonist action, nor does it act on metabolic pathways , serotonin receptors , or dopamine receptors .
Answer. d
Copyright 2014 Delhi Academy of Medical Sciences, All Rights Reserved. 59/88

(138). Which of the following antifungal drugs is strongly associated with drug interactions resulting from inhibition of CYP 3A4?
a. Amphotericin B
b. Caspofungin
c. Fluconazole
d. Ketoconazole

Solution. Ans 138: (d) Ketoconazole
Reference Read the text below
Sol:
Ketoconazole is a potent inhibitor of the cytochrome P450 enzyme, CYP 3A4. Because CYP 3A4 is involved in the metabolism of
almost 50% of the drugs in use, ketoconazole is implicated in many pharmacokinetic drug interactions.
Amphotericin B causes nephrotoxicity and may therefore interfere with the excretion of other drugs.
Caspofungin appears to be free of most drug interactions.
Fluconazolealthough a member of the same (azole) group as ketoconazole,is much less potent as a CYP 3A4 inhibitor and is
associated with far fewer drug interactions.
Flucytosineis associated with few pharmacokinetic drug interactions,but by metabolism to fluorouracil it is toxic to the bone marrow
and other rapidly dividing cells.
Answer. d

(139). All of the following drugs are recommended for treatment of beta blocker induced excessive bradycardia and/or decrease in cardiac
output except :-
a. Dopamine
b. Dobutamine
c. Glucagon
d. Phentolamine

Solution. Ans-139: (d) Phentolamine
Ref:Read the text below
Sol:
Drugs useful in b blocker toxicity/Bradycardia:
Atropine
Isproterenol
Glucagon
Dopamine & Dobutamine
Epinephrine/norepinephrine
Answer. d

(140). Which of the following antibiotics should typically be administered to prevent bacterial endocarditis in patients with cardiac valve
disease who are undergoing dental surgery ?
a. Ciprofloxacin
b. Amoxicillin
c. Ceftriaxone
d. Doxycycline

Solution. Ans-140: (b) Amoxicillin
Ref:Read the text below
Sol:
Oral amoxicillin should be administered to cardiac valve patients one hour before and 6 hours after dental surgery in order to
prevent endocarditis resulting from surgical bacteremia.
Gram-positive organisms sensitive to penicillins are the most likely cause of endocarditis in this setting.
Erythromycin should be administered to most penicillin-allergic patients, but high-risk penicillin-allergic patients should receive
vancomycin.
Ampicillin or clindamycin may be given parenterally to low-risk patients who cannot take oral amoxicillin.
Answer. b
Copyright 2014 Delhi Academy of Medical Sciences, All Rights Reserved. 60/88

(141). Which drug should be used cautiously in a pregnant female, as it causes foetal abnormalities and potential risk clearly outweighs
possible benefits:
a. Ergometrine
b. Valproate
c. Thyroxine
d. Aspirin

Solution. Ans-141: (a) Ergometrine
Ref:Read the text below
Sol:
RISK CATEGORY OF DRUGS DURING PREGNANCY
Answer. a

(142). The treatment of choice for "Familial Mediterranean fever" is :-
a. Cyclophosphamide
b. Cyclosporine
c. Quinine
d. Colchicine

Solution. Ans-142: (d) Colchicine
Ref:Read the text below
Sol:
The treatment of choice for Familial Mediterranean fever (FMF) is daily oral co1chicine,which decreases the frequency and
intensity of attacks and prevent the development of amyloidosis in compliant patients.
Intermittent dosing at the onset of attacks is not as effective as daily prophylaxsis and is of unproven value in preventing
amyloidosis.
The usual adult dose of colchicine is 1.2 to 1.8 mg/d, which causes substantial reduction in symptoms in two-thirds of patients
Common side effects of colchicine include bloating, abdominal IPS, lactose intolerance, and
diarrhea. They can be minimized by starting at a low dose and gradually advancing as tolerated, splitting the dose, use of
simethicone for flatulence, and avoidance of dairy products .
lf taken by either parent at the time of conception, colchicine may cause a small increase in the risk of trisomy 21 (Down
syndrome)
Answer. d

Copyright 2014 Delhi Academy of Medical Sciences, All Rights Reserved. 61/88
(143). All of the following drugs are contraindicated in patients with G-6-PD deficiency, except:
a. Phenacetin
b. Vitamin K
c. Nalidixic acid
d. Ceftriaxone

Solution. Ans-143: (d) Ceftriaxone
Ref:Read the text below
Sol:
Answer. d

(144). 50 to 80% patients of glucagonomas present with metastasis,so curative surgical resection is not possible, so drug like long acting
somatostatin analogues such as octreotide are beneficial. Sign and symptoms not responding to octerotide is:
a. Skin rash
b. Weight loss
c. Diarrhoea
d. Glucose intolerance

Solution. Ans-144: (d) Glucose intolerance
Ref:Read the text below
Sol:
In 50 to 80% of patients metastasis are present at presentation, so curative surgical resection is not possible.
Surgical debulking in patients with advanced disease or other antitumor treatments may be beneficial.
Long-acting somatostatin analogues such as octrebtide or lanreotide improve the skin rash in
75% of patients
May improve the weight loss, pain, and diarrhoea
Usually do not improve the glucose intolerance
Answer. d

Copyright 2014 Delhi Academy of Medical Sciences, All Rights Reserved. 62/88
(145). Medical treatment of hypercalcemia varies with its severity. Drugs like diuretics, etidronate, glucocorticoids,calcitonin etc are
beneficil:drug associated with rapid tachyphylaxis is:
a. Diuretics
b. Etidronate
c. Glucocorticoids
d. Calcitonin

Solution. Ans-145: (d) Calcitonin
Ref:Read the text below
Sol:
Calcitonin rapidly lowers serum calcium, although usually not to normal concentrations. However, despite continued
administration of calcitonin, tachyphylaxis usually develops in 48 to 72 hours.
Tachyphylaxis may be causedby down-regulation of calcitonin receptors or postreceptor desensitization.
Nevertheless, calcitonin acts rapidly, is safe, and is often used in the initial treatment of hypercalcemia, together with other more
effective drugs.
Answer. d

(146). Contra-indicators to -blockers include all of the following except
a. Raynaud's disease
b. Wolff-Parkinson-White syndrome
c. Chronic obstructive pulmonary disease
d. Intermittent claudication

Solution. Ans 146 : (b)Wolff-Parkinson-White syndrome
Ref:Read the text below
Sol:
The three classic groups of patients where -blockers are contra-indicated are
Peripheral vascular disease and Raynaud's
Asthma and COPD
Heart failure
As well as blockade of -receptors, some of these drugs may have additional features such as cardioselectivity, partial agonist
activity and membrane-stabilising activity.
Cardioselective -blockerssuch as acebutalol, atenolol, bisoprolol, metoprolol and esmolol block the
1
-receptors to a greater degree
than the
2
-receptors.
Some of the -blocking drugs e.g. acebutalol and oxprenolol have partial agonist activity, sometimes described as intrinsic
sympathomimetic activity (ISA). It has been suggested that drugs with this property have a lower incidence of the undesirable side-
effects such as bradycardia, heart failure, cold extremities and bronchospasm.
Propranolol has membrane-stabilising activity, potent local anaesthetic properties and demonstrates class II anti-
arrhythmic properties.
Answer. b

(147). Drug combinations that have no significant clinical interaction include
a. Trimethoprim an phenytoin
b. Etomidate and hydrocortisone
c. Rifampicin and propofol
d. Ciprofloxacin and aminophylline

Solution. Ans 147 : (b)Etomidate and hydrocortisone
Ref:Read the text below
Sol:
Trimethoprim and phenytoininterfere with folate metabolism causing megaloblastic anaemia.
Rifampicinis a liver enzyme inducer which will increase the metabolism of drugs metabolised by the liver.
Ciprofloxacin and erythromycin are liver enzyme inhibitors which will decrease the metabolism of drugs metabolised by
the liver.
Answer. b
Copyright 2014 Delhi Academy of Medical Sciences, All Rights Reserved. 63/88

(148). Tramadol
a. Is a controlled drug
b. Has affinity for binding at the mu opioid receptor comparable to that of morphine
c. Acts predominantly by inhibiting the reuptake of noradrenaline and serotonin (5-HT)
d. May be used concurrently with a MAOI

Solution. Ans 148 : (c)Acts predominantly by inhibiting the reuptake of noradrenaline and serotonin (5-HT)
Ref:Read the text below
Sol:
Tramadol is not a controlled drug. It has a weak affinity for opioid receptors.
The affinity for binding at the mu receptor is 2.1 mM compared with 0.0003 M for morphine.
Its main mode of action is the modulation of central monoaminergic pathways.
As with pethidine (and possibly other opioids), the use of tramadol with MAOI is contraindicated. Both drugs delay the removal
of 5-HT from its site of action.
The interaction may presents clinically as sudden agitation, unmanageable behaviour, headaches, hypertension or hypotension,
rigidity, hyperpyrexia, convulsions and coma
Answer. c

(149). The metabolism of which of the following does not follow zero-order kinetics in clinical doses
a. Phenytoin
b. Aspirin
c. Ethanol
d. Propranolol

Solution. Ans 149 : (d)Propranolol
Ref:Read the text below
Sol:
The metabolism of the following observe zero-order kinetics
Phenytoin
Aspirin
Ethanol
Paracetamol (in toxic doses)
Thiopentone (in toxic doses)
Answer. d

Copyright 2014 Delhi Academy of Medical Sciences, All Rights Reserved. 64/88
(150). The following are examples of hepatic enzyme inducer
a. Cimetidine
b. Erythromycin
c. Griseofulvin
d. Amiodarone

Solution. Ans 150 : (c)Griseofulvin
Ref:Read the text below
Sol:
ENZYME INDUCERS AND INHIBITORS IN CLINICAL USE
Inducers Inhibitors
Barbiturates
Carbamazepine
Ethanol (chronic)
Inhalational anaesthetics
Griseofulvin
Phenytoin
Primidone
Rifampicin
Amiodarone
Cimetidine
Ciprofloxacin
Dextropropoxyphene (co-proxamol)
Ethanol (acute)
Etomidate
Erythromycin
Fluconazole
Ketoconazole
Metronidazole
Answer. c

(151). Hepatic first-pass metabolism
a. Is avoided by giving the drug transdermally
b. Is avoided by giving the drug sublingually
c. Is avoided by giving the drug rectally
d. Is seen when a drug has a high hepatic extraction ratio

Solution. Ans 151 : (c)Is avoided by giving the drug rectally
Ref:Read the text below
Sol:
Transdermal, sublingual and parenteral(IV, IM, SC) routes avoid the hepatic portal circulation and hence the hepatic first pass
metabolism.
A variable proportion of the drug given rectally will be absorbed into the portal circulation, therefore first pass metabolism is
not totally avoided.
Drugs with high hepatic extractionratios undergo substantial first pass metabolism
Answer. c

Copyright 2014 Delhi Academy of Medical Sciences, All Rights Reserved. 65/88
(152). Select incorrect statement regarding "First-order kinetics"
a. A constant proportion of drug metabolised in a given time period
b. The absolute amount eliminated is greatest when plasma concentration is greatest
c. The rate of elimination and elimination half-life is constant, irrespective of plasma concentration
d. The enzyme responsible for the reaction is saturated

Solution. Ans 152 : (d)The enzyme responsible for the reaction is saturated
Ref:Read the text below
Sol:
Zero-order kinetics are observed when the enzyme responsible for the reaction is saturated and when the reaction is
represented by a linear relationship.
Summary of the differences between zero-order and first-order kinetics:
Zero-order:
Absolute amount eliminated is the same, regardless of plasma concentration;
Rate of elimination varies with the plasma concentration;
Constant amount of drug metabolised per unit time;
Reflects saturation of enzyme: occurs when plasma concentration exceeds capacity of enzyme.
First-order:
Constant proportion of drug metabolised in a given time period;
Absolute amount eliminated is greatest when plasma concentration is greatest;
Rate of elimination and elimination half-life are constant, irrespective of concentration.
Answer. d

(153). Which of the following is useful in rheumatoid arthritis because it binds tumor necrosis factoralpha (TNF-alpha)?
a. Leflunomide
b. Infliximab
c. Methotrexate
d. Paclitaxel

Solution. Ans 153: (b) Infliximab
Reference Read the text below
Sol:
Infliximab is a chimeric monoclonal antibody that binds TNF-alpha and is useful in rheumatoid arthritis, ulcerative colitis, and
several other immune diseases.
Leflunomide inhibits ribonucleotide synthesis and thereby reduces T-cell proliferation.
Methotrexate is an antimetabolite that inhibits cell proliferation. It is useful in rheumatoid arthritis.
Paclitaxelis a microtubule-binding agent that is cytotoxic in certain neoplasms and slows restenosis in coronary stents. It is not used
in autoimmune diseases.
Answer. b

Copyright 2014 Delhi Academy of Medical Sciences, All Rights Reserved. 66/88
(154). Which of the following mechanisms most accurately describes the blocking action of methimazole?
a. Iodine uptake by the thyroid
b. Organification of iodine in the thyroid
c. Peripheral conversion of T4 to T3
d. Peripheral thyroxine (T4) and T3 receptors

Solution. Ans 154: (b) Organification of iodine in the thyroid
Reference Read the text below
Sol:
Methimazole (and other thioamides) act primarily by inhibiting thyroid peroxidase, the critical enzyme in iodine organification.
They also inhibit coupling of iodotyrosines in thyroglobulin. They do not block iodine uptake by the gland.
Block of iodineuptake by the thyroid is a property of antithyroid anions such as thiocyanate (SCN) and perchlorate (ClO4 ).
Inhibition of peripheral conversion of T4 to T3 isthe mechanism of beta blockers and iodine-containing radio-opaque drugs such
as ipodate.
Block of peripheral thyroid receptors is not a mechanism of any currently available drug. Block of thyroglobulin proteolysis and
reduced release of thyroxine from the gland is a property of iodide ion.
Answer. b

(155). Which of the following adverse effects of chronic high-dose prednisone administration represents a mineralocorticoid effect?
a. Buffalo hump
b. Easy bruising
c. Edema
d. Moon facies

Solution. Ans 155: (c) Edema
Reference Read the text below
Sol:
Edema and hypertensionwill occur with any corticosteroid that has mineralocoid properties if given in high dosage over a long
period.
Prednisone, though much less potent than cortisol as a mineralocorticoid, retains some salt-retaining potency. Newer
glucocorticoids, such as dexamethasone and triamcinolone,are much less likely to retain salt and water.
Buffalo hump and moon facies reflect the fat metabolism effects of glucocorticoid activity. Easy bruising and striae reflect the
catabolic action of glucocorticoids on proteins.
The mechanism of central nervous system dysfunction is not understood, but it is associated with glucocorticoid, not
mineralocorticoid, activity.
Answer. c

(156). Which of the following drugs is considered a first-line agent in the chronic treatment of rheumatoid arthritis and in the treatment of
choriocarcinoma?
a. Cyclosporine
b. Hydroxychloroquine
c. Methotrexate
d. Sulfasalazine

Solution. Ans 156: (c) Methotrexate
Reference Read the text below
Sol:
Methotrexate in very low dosage has proven effective in slowing the progression of rheumatoid arthritis and has low toxicity. It is
also one of the most effective (in much higher dosage) antimetabolite cancer chemotherapeutic drugs used in choriocarcinoma.
Cyclosporineis not used in either condition but is a first-line drug in transplant immunopharmacology.
Hydroxychloroquine is a disease-modifying antirheumatoid arthritis drug, but has no efficacy in the treatment of cancer. It is more
toxic and less effective in rheumatoid arthritis than methotrexate.
Sulfasalazine is also effective in slowing the progression of joint damage in rheumatoid arthritis but has no efficacy in
cancer.
Answer. c
Copyright 2014 Delhi Academy of Medical Sciences, All Rights Reserved. 67/88

(157). Glucocorticoids have been shown to be of benefit for treatment for all of the following infections except
a. Aspergillus fumigatus pneumonia
b. Mycobacterium tuberculosis pericarditis
c. Pneumocystis carinii pneumonia
d. severe typhoid fever

Solution. Ans 157: (a) Aspergillus fumigatus pneumonia
Reference Read the text below
Sol:
The role of immune modulators in serious infections has received increased attention in recent years as understanding of the
cytokine and inflammatory systems have evolved.
In several models of disease, an aggressive host anti-inflammatory response may increase organ damage.
Steroids are useful adjunctsfor several infections as antibiotics alone can increase inflammation and cytokine release due to lysis of
intact organism and release of pro-inflammatory intracellular content.
However, mold infections, such as Aspergillus, are apt to worsen in the setting of glucocorticoids.
Answer. a

(158). Only bisphosphonate currently approved for intravenous use in 'paget disease'.
a. Etidronate
b. Pamidronate
c. Alendronate
d. Risedronate

Solution. Ans-158: (b) Pamidronate
Ref:Read the text below
Sol:
BISPHOSPHONATES
The bisphosphonates are analogues of pyrophosphate, with high affinity for bone, especially in areas of increased bone turn over,
where they are powerful inhibitors of bone resorption.
The bisphosphonates are concentrated in areas of high bone turn over and are taken up by and inhibit osteoclast action.
The potency of the compounds for inhibition of bone resorption varies a thousandfold, increasing in the order of etidronate,
tiludronate, pamidronate, alendronate, and risdronate.
Pamidronate is the only bisphosphanate currently approved for intravenous use in paget disease.
Answer. b

Copyright 2014 Delhi Academy of Medical Sciences, All Rights Reserved. 68/88
(159). The goal in treating type II diabetes is to maintain blood glucose concentrations within normal limits : drug producing hypoglycemic
reaction is:
a. Glyburide
b. Metformin
c. Glucagon
d. Acarbose

Solution. Ans-159: (a) Glyburide
Ref:Read the text below
Sol:
Answer. a

(160). Aspirin and ticlopidine share which of the following characteristics ?
a. Both may cause reversible agranulocytosis
b. Both inhibit adenosine diphosphate (ADP)- induced platelet aggregation.
c. Both inhibit cyclooxygenase and the formation of thromboxane
d. Both can be used to prevent stroke and myocardial Infarction

Solution. Ans-160: (d) Both can be used to prevent stroke and myocardial Infarction
Ref:Read the text below
Sol:
Both aspirin and ticlopidine can be used to prevent vascular events that result from the formation of arterial thrombi in patients at
risk.
Ticlopidine may cause leukopenia, so the white blood cell count must be monitored periodically.
Ticlopidine acts by inhibiting ADP-inducedplatelet aggregation, whereas aspirin inhibits the synthesis of thromboxane, another
potent platelet derived aggregation factor.
Neither aspirin nor ticlopidine is particularly effective in preventing venous thrombosis, and heparin or warfarin should be used
for that purpose.
Answer. d

Copyright 2014 Delhi Academy of Medical Sciences, All Rights Reserved. 69/88
(161). Which of the following agents used in treating testicular cancer, requires hydration and dieresis to prevent renal toxicity?
a. Bleomycin
b. Cisplatin
c. Cyclophosphamide
d. 5-fluorouracil

Solution. Ans-161: (b) Cisplatin
Ref:Read the text below
Sol:
Cisplatin binds to DNA where it forms intra- and interstrand crosslinks. Cisplatin is particularly effective in testicular and ovarian
cancers in combination with other antitumor agents.
Cisplatin exerts a renal toxicity that may be prevented by the infusion of saline to maintain a high urine flow.
Ototoxicity involving high-frequency hearing loss is an effect that is not prevented by hydration. The natural product bleomycin
binds to DNA and causes single- and double-strand breaks, leading to cytotoxicity. The drug is particularly useful against Hodgkin
lymphoma and testicular tumors.
Bleomycin has the serious toxicity of pulmonary fibrosis. Cyclophosphamide is widely used in combination regimens.
Nausea and vomiting are the most common toxicities. Hemorrhagic cystitis may be minimized by hydration and use of the drug mesna.
Note that this toxicity is not at the level of the kidney.
The pyrimidine analog 5-fluorouracil (5-FU) is used to treat a wide variety of carcinomas. Toxicity from 5-FU is expressed
as GI disturbances (anorexia, nausea, stomatitis, and diarrhea) and myelosuppression.
Answer. b

(162). Which of the following chemotherapeutic agents' mechanisms of action involve inhibition of thymidylate synthase and incorporation into
RNA?
a. Anastrozole
b. Cytarabine
c. Doxorubicin
d. 5-fluorouracil

Solution. Ans-162: (d) 5-fluorouracil
Ref:Read the text below
Sol:
The pyrimidine analog 5-fluorouracil (5-FU) is metabolized by ribosylation and phosphorylation to the nucleotide level (F-
UMP).F-UMP is further metabolized to F-dUMP, an inhibitor of thymidylate synthase.
Cells then become starved for TTP and incorporate FdUTP and dUTP in its place in DNA. 5-FU also becomes incorporated in RNA,
leading to inhibition of RNAprocessing. Anastrozole is a nonsteroidal inhibitor of aromatase, an enzyme required for synthesis
of estrogens.
This drug is useful against advanced estrogen or progesterone receptor-positive breast cancer.Cytarabine (cytosine arabinoside,
ara-C), is an S-phase-specific antimetabolite that is metabolized to the triphosphate form, which blocks DNA synthesis. It is used
exclusively in acute myelogenous leukemia.
Doxorubicin is an anthracycline antibiotic that intercalates into DNA causing strand breakage and blockage of both DNA and RNA
synthesis.This agent is widely used in combination regimens for breast, endometrial, ovarian, testicular, and thyroid carcinomas,
and several sarcomas and lymphomas.
Imatinib is an inhibitor of the tyrosine kinase activity of the Bcr-Abl oncogene product. It is a drug of choice in chronic myelogenous
leukemia with the Philadelphia chromosome translocation.
Answer. d

Copyright 2014 Delhi Academy of Medical Sciences, All Rights Reserved. 70/88
(163). Which of the following chemotherapeutic agents' mechanisms of action involves inhibition of topoisomerase II and results in DNA strand
breakage?
a. Dacarbazine
b. Etoposide
c. Lomustine
d. Prednisone

Solution. Ans-163: (b) Etoposide
Ref:Read the text below
Sol:
Etoposide is a semisynthetic derivative of podophyllotoxin, a constituent of the mandrake plant.
Etoposide is an inhibitor of topoisomerase II, an enzyme that relaxes supercoiled DNA by breaking one strand and passing the
second strand through the break before closing the break.
Etoposide inhibits the closure step and results in an accumulation of DNA strand breaks, leading to cell death.
Etoposide is used to treat testicular tumors and small cell carcinoma of the lung in combination with cisplatin.
Leukopenia is the dose-limiting toxicity seen with this drug.
Answer. b

(164). A13-year-old boy with severe asthma has been treated for 6 months. His parents now complain that he seems to be very fat around the
face and shoulders, bruises easily, and has discolored streaks on his abdomen. Laboratory tests reveal moderate hyperglycemia. He has
probably been receiving ?
a. Ipratropium
b. Prednisone
c. Salmeterol
d. Terbutaline

Solution. Ans-164: (b) Prednisone
Ref:Read the text below
Sol:
This patients presentation is typical of glucocorticosteroid toxicity (iatrogenic Cushing syndrome).
Prednisone is an effective corticosteroid frequentlyused by mouth to control severe asthma that is not responsive to inhaled
corticosteroids.
High doses of prednisone, given chronically, very predictably cause signs and symptoms of hypercorticism.
Ipratropium is an antimuscarinicagent used in asthma by inhalation.
Answer. b

(165). Which of the following describes a possible adverse effect of long-term chronic treatment of schizophrenia with a phenothiazine such as
fluphenazine?
a. Diarrhea, nausea, and vomiting
b. Reduced secretion of prolactin
c. Tardive dyskinesia
d. Tourette syndrome

Solution. Ans-165: (c) Tardive dyskinesia
Ref:Read the text below
Sol:
Long-term treatment of schizophrenia with potent dopamine D2 receptor antagonists is associated with a high incidence of
the irreversible extrapyramidal dystonias called tardive dyskinesia.
The phenothiazines (and haloperidol) are D2 receptor antagonists in the CNS; their success in the treatment of schizophrenia
resulted in the hypothesis that the antipsychotic effect requires D2 blockade.
Data from newer, atypical antipsychotic agents casts some doubt on this hypothesis because drugs, such as clozapine and risperidone,
have a low affinity for D2 receptors
Answer. c

Copyright 2014 Delhi Academy of Medical Sciences, All Rights Reserved. 71/88
(166). The initial and crucial event that enables glyburide to cause the pancreatic cells to release insulin is
a. Increased potassium (K) efflux
b. Binding to receptors on the adenosine triphosphate (ATP)-sensitive K+ channels
c. Closing of voltage-dependent Ca channels
d. Decreased phosphorylation reactions

Solution. Ans-166: (b) Binding to receptors on the adenosine triphosphate (ATP)sensitive K+ channelsRef: Read the text
below.
Sol:
Glyburide is an oral hypoglycemic agentthat is classified as a sulfonylurea derivative. This compound is used in the treatment of
NIDDM.
For hypoglycemic action, glyburide needs functional cells in the pancreas, because it is ineffective in depancreatized or severely
insulin-deficient patients.
Sulfonylurea compounds stimulate the release of insulin from the pancreas by a proposed mechanism of action involving the initial
binding of the drug to a receptor on the ATP-sensitive K channels in the cell.
As a consequence of this drugreceptor interaction, there is an inhibition of K efflux from the cell, which then produces
depolarization of the membrane.
The depolarization of the membrane opens voltage-dependent Ca channels to allow the entrance of Ca into the cell. The increased Ca
concentration stimulates phosphorylation reactions, followed by the process of exocytosis, which causes the release of
insulin from the cells.
Other drugs, such as diazoxide and epinephrine, reduce insulin secretion by causing hyperpolarization of the cell,
decreasing Ca ion influx, and thereby preventing the process of exocytosis for the release of insulin.
Answer. b

(167). The treatment of myxedema coma can include which of the following agents?
a. Thyroglobulin
b. Levothyroxine
c. Lithium
d. Propylthiouracil (PTU)

Solution. Ans-167: (b) Levothyroxine
Ref: Read the text below.
Sol:
Myxedema coma is a medical emergency and should be treated as soon as the diagnosis is established.
Treatment involves the use of several drugs to correct this condition. It appears that the selection of either LT4, liothyronine, or
liotrix is appropriate.
LT4, however, is the drug of choice.Supportive treatment of symptoms is also indicated. Maintenance of respiration and
administration of fluids and electrolytes, along with glucose if hypoglycemia is diagnosed, should be provided.
Because adrenal insufficiency may be present, administration of glucocorticoids is initially recommended. Thyroglobulin, a protein of
high molecular weight, is a component of the thyroid gland. Although preparations are available, this drug is not indicated
in myxedema coma.
Protirelin is a synthetic thyrotropin-stimulating hormonethat is used in the diagnosis of thyroid function.
Although lithium was once tested as a drug to treat hyperthyroidism because it induced hypothyroidism, lithium has no place in the
therapy of hyperthyroidism. In addition, PTU is an antithyroid drug used in the management of hyperthyroidism.
Answer. b

Copyright 2014 Delhi Academy of Medical Sciences, All Rights Reserved. 72/88
(168). A patient develops hyperglycemia, hyperuricemia, and hypomagnesemia on which of the following diuretic agents?
a. Hydrochlorothiazide
b. Triamterene
c. Spironolactone
d. Acetazolamide

Solution. Ans-168: (a) Hydrochlorothiazide
Ref: Read the text below.
Sol:
The thiazide diuretics and the loop diuretics have a number of adverse reactions in common.
Hydrochlorothiazide and the loop diuretics cause hyperglycemia by possibly reducing the secretion of insulin from the
pancreas.
Because these drugs can elevate blood levels ofglucose, they should be used with caution when administered to patients with
diabetes mellitus.
The development of hyperuricemia as a consequence of the use of hydrochlorothiazide or loop diuretics is related to the fact that
these drugs interfere with the proximal tubule secretion of uric acid because of volume depletion.
The alteration in serum Mg (hypomagnesemia) is caused by both hydrochlorothiazide and the loop diuretics, by blocking the
reabsorption of Mg.
Answer. a

(169). Of the following, which adverse reaction is not associated with furosemide?
a. Hyperglycemia
b. Tinnitus
c. Hypotension
d. Metabolic acidosis

Solution. Ans-169: (d) Metabolic acidosis
Ref: Read the text below.
Sol:
The loop, or high-ceiling,diuretics furosemide and ethacrynic acid are cleared by the kidney with such celerity that even high
doses repeatedly administered do not result in significant accumulation.
Chronic administration of these agents, however, may lead to alkalosis with hyponatremia in association with rapid removal of
edema fluid.
Other toxic manifestations of loop diureticsinclude fluid and electrolyte imbalance, gastrointestinal symptoms, interstitial
nephritis, hyperglycemia, tinnitus, and infrequent, but serious, ototoxicity.
Besides being used as a diuretic agent, furosemide is used in the treatment of hypertension.
Answer. d

(17
0).
Cholesterol gallstones may be dissolved by oral treatment with
a. Dehydrocholic acid
b. Methyl tertiary butyl ether
c. Chenodeoxycholic acid
d. Monoctanoin

Solution. Ans-170: (c) Chenodeoxycholic acid
Ref: Read the text below.
Sol:
Chenodeoxycholic acid (chenodiol) and ursodiolhave proved
to be effective in some patients with cholesterol gallstones.
Lovastatin lowersblood cholesterol levels but has no effect on
gallstones.
Methyl tertiary butyl ether and a new agent, monoctanoin,
are infused directly into the common duct and will dissolve
gallstones.
Answer. c
Copyright 2014 Delhi Academy of Medical Sciences, All Rights Reserved. 73/88

(171). Which of the following vitamins in large doses is teratogenic?
a. Vitamin A
b. Vitamin B12
c. Vitamin C
d. Vitamin D

Solution. Ans-171: (a) Vitamin A
Ref: Read the text below.
Sol:
Pregnant women should not take more than a 25% increase in the normal dietary intake of vitamin A,because it is definitely
teratogenic, especially in the first trimester of pregnancy.
Great caution is to be taken in premenopausal females in the therapy of acne and skin wrinkling in which tretinoin or isotretinoin
is the therapeutic agent.
None of the other vitamins is particularly teratogenic, except perhaps vitamin D.
Answer. a

(172). Which of the following psychotropic drugs must be monitored for hematotoxic effects?
a. Buspirone
b. Clozapine
c. Haloperidol
d. Lithium carbonate

Solution. Ans-172: (b) Clozapine
Ref:Read the text below
Sol:
Clozapine causes agranulocytosisin a small but consistent fraction of patients; monitoring is mandatory.
Buspirone is an antianxietyagent with minimal sedative action.
Haloperidol is an older, highly potent antipsychotic drug used in schizophrenia.
Lithium carbonateis an important antimanic drug. It apparently acts by interfering with inositol phosphate cycling and second
messenger synthesis in neurons.
Answer. b

(173). Which of the following is used in treating major depression and has the greatest sedative effect?
a. Bupropion
b. Desipramine
c. Fluoxetine
d. Mirtazapine

Solution. Ans-173: (d) Mirtazapine
Ref:Read the text below
Sol:
All of the agents listed are useful antidepressants but only mirtazapine has significant sedative effects.
Answer. d

Copyright 2014 Delhi Academy of Medical Sciences, All Rights Reserved. 74/88
(174). Which of the following is the drug of choice to reverse bronchoconstriction in an acute asthma attack?
a. Albuterol
b. Aminophylline
c. Cromolyn
d. Ipratropium

Solution. Ans-174: (a) Albuterol
Ref:Read the text below
Sol:
Inhaled albuterol (or other beta-2-selective receptor agonists including bitolterol, metaproterenol, pirbuterol, and terbutaline) are
the usual agents of choice for treating bronchoconstriction in an acute asthma attack.
These beta-2-selective agonists produce bronchial relaxation by stimulating cyclic AMP (cAMP) formation in bronchiolar
smooth muscle and cause less tachycardia than nonselective beta agonists.
Unfortunately, they do cause some tachycardia and skeletal muscle tremor.
Aminophylline and other methylxanthines are rarely used to terminate acute episodes of asthma because they must be
administered parenterally for rapid onset of effect.
Answer. a

(175). Which of the following drugs is useful in treatment of gout with recurrent renal urate stones because it decreases the excretion of uric
acid?
a. Allopurinol
b. Aspirin
c. Colchicine
d. Indomethacin

Solution. Ans-175: (a) Allopurinol
Ref:Read the text below
Sol:
Allopurinol and its metabolite alloxanthine inhibit xanthine oxidase, thus preventing conversion of xanthine and
hypoxanthine to uric acid.
Although xanthine and hypoxanthine then accumulate, these compounds are more soluble than uric acid and less likely to deposit in
joints or precipitate in the urine.
Most doses of aspirin (choice B) increase retention of uric acid, especially low doses.
Colchicine is an inhibitorof microtubule function that brings relief in an acute gout attack by inhibiting the motility of granulocytes
and preventing the formation of mediators of inflammation by leukocytes.
Because of its toxicity at higher doses, it is now used chiefly at low doses to prevent acute attacks.
Answer. a

Copyright 2014 Delhi Academy of Medical Sciences, All Rights Reserved. 75/88
(176). Which of the following drugs can be used in rheumatoid arthritis with the lowest probable incidence of GI complications?
a. Aspirin
b. Celecoxib
c. Ibuprofen
d. Misoprostol

Solution. Ans-176: (b) Celecoxib
Ref:Read the text below
Sol:
A through C and E are NSAIDs. NSAIDs have long been drugs of first choice in arthritis treatment. Their primary mechanism of
action in arthritis appears to be inhibition of COX, an enzyme required for the synthesis of inflammatory and other prostaglandins.
Two forms of COX are present in the body: COX-1, which is required for synthesis of several useful prostaglandins (e.g., PGE1, a
cytoprotective agent in the stomach), and COX-2, the isoform responsible for synthesis of prostacyclin as well as most of the damaging
prostaglandins.
Celecoxib is more selective for COX-2 and thus has a lower incidence of adverse GI effects. The older NSAIDs (choices A, C, and E)
inhibit both
COX-1 and COX-2 with less selectivity and thus reduce protective prostaglandins, resulting in a high incidence of GI
disorders, especially peptic ulceration.
Answer. b

(177). Which of the following agents would cause the greatest increase in heart rate in a functioning transplanted heart?
a. Amphetamine
b. Cocaine
c. Ephedrine
d. Isoproterenol

Solution. Ans-177: (d) Isoproterenol
Ref:Read the text below
Sol:
The transplanted heart lacks functional innervation for at least 2 years after surgery, and possibly longer.
Because the nerves are cut in the procedure, nerve endings degenerate (Wallerian degeneration) and transmitter stores are lost.
Adrenoreceptors on the denervatedmyocardial cells are normal or even increased in sensitivity, so responses to direct-acting
betaadrenoreceptor agonists such as isoproterenol are retained.
Indirectly acting sympathomimetics, on the other hand, are relatively ineffective because they act through the release or
amplification of endogenous norepinephrine
Answer. d

(178). Which of the following agents produces vasodilation by increasing nitric oxide synthesis in endothelial cells?
a. Diazoxide
b. Histamine
c. Minoxidil
d. Nitroprusside

Solution. Ans-178: (b) Histamine
Ref:Read the text below
Sol:
Histamine activates nitric oxide synthase by binding to H1 receptors in the endothelium. Nitric oxide synthesis is increased, and
this molecule rapidly diffuses into the adjoining smooth muscle, where it causes vasodilation.
Diazoxide is a powerful vasodilator used in hypertensive emergencies. It acts by opening potassium channels in vascular smooth
muscle and hyperpolarizing these cells.
Minoxidil another vasodilator used in severe hypertension, is converted to the sulfate metabolite, which similarly opens potassium
channels and hyperpolarizes vascular smooth muscle.
Nitroprusside contains nitric oxide that is spontaneously released in the blood. Stimulation of endothelial synthesis is not
involved.
Answer. b
Copyright 2014 Delhi Academy of Medical Sciences, All Rights Reserved. 76/88

(179). Which of the following statements correctly associates a CNS drug with its mechanism of action?
a. Bupropion-activation of endocannabinoid receptors
b. Diazepam-facilitation of GABAstimulated chloride channel opening
c. Fluoxetine-selective inhibition of presynaptic norepinephrine reuptake
d. Pentobarbital-inhibition of NMDA receptors

Solution. Ans-179: (b) Diazepamfacilitation of GABAstimulated chloride channel opening
Ref:Read the text below
Sol:
The benzodiazepine agents, including diazepam, facilitate the actions of the inhibitory neurotransmitter GABA, which acts
on GABAA receptors to open chloride ion channels.
Bupropion is probablyan inhibitor of norepinephrine and dopamine uptake anddoes not act on GABA receptors.
Fluoxetine is a selective inhibitor of serotonin uptake.
Pentobarbital is a modulator of the same GABA-sensitive chloride channel affected by benzodiazepines, although its mechanism of
action is slightly different.
Tranylcypromine is an inhibitor ofMAO rather than catechol-O-methyltransferase (COMT).
Answer. b

(180). Which of the following agents is associated with numerous drug-drug interactions because of its inhibition of hepatic cytochrome P450
activity?
a. Atracurium
b. Cromolyn
c. Ketoconazole
d. Phenobarbital

Solution. Ans-180: (c) Ketoconazole
Ref:Read the text below
Sol:
Ketoconazole is a potent inhibitor of cytochrome P450 isozymes; its antifungal action is the result of inhibition of fungal
P450.Atracurium (choice A) is used to produce skeletal muscle relaxation during surgery.
Atracurium is eliminated by a spontaneous chemical reaction and by plasma cholinesterases, and is not a substrate or inhibitor of
cytochrome P450.
Cromolyn sodium is not a substrate or inhibitor of cytochrome P450.
The extremely small amounts of this drug that are absorbed are excreted unchanged in the urine and bile.
Phenobarbital is eliminated by cytochrome P450 metabolism, and is an inducer of drug metabolism activity.
Answer. c

(181). A cell cycle specific anticancer drug that acts mainly in the M phase of the cycle is
a. Cisplatin
b. Etoposide
c. Methotrexate
d. Pacilitaxel

Solution. Ans-181: (d) Paclitaxel
Ref: KDT--830
Sol:
Vinca alkaloids (vincristine, vinblastine and vinorelbine) and taxanes (paclitaxel and docetaxel) act in M-phase of cell cycle.
Vinca alkaloids inhibitits the formation whereas taxanes inhibit the breakdown of mitotic spindle.
Answer. d

Copyright 2014 Delhi Academy of Medical Sciences, All Rights Reserved. 77/88
(182). Drug causing hemorrhagic cystitis
a. Cyclophosphamide
b. 5 FU
c. Methotrexate
d. Cytarabine

Solution. Ans 182: (a) Cyclophosphamide
Ref:Read the text below
Sol:
Cyclophosphamide and ifosfamide
The most prominent toxicities of both drugs (after alopecia, nausea, vomiting, and diarrhea) are bone marrow depression,
especially leukocytosis, and hemorrhagic cystitis, which can lead to fibrosis of the bladder.
The latter toxicity has been attributed to acrolein in the urine in the case of cyclophosphamide and to toxic metabolites of ifosfamide.
[Note: Adequate hydration as well as IV injection of MESNA (sodium 2-mercaptoethane sulfonate), which neutralizes the
toxic metabolites, minimizes this problem.]
Other toxicities include effects on the germ cells, resulting in amenorrhea, testicular atrophy, aspermia, and sterility.
Veno-occlusive disease of the liver is seen in about 25 percent of the patients.
A fairly high incidence of neurotoxicity has been reported in patients on high-dose ifosfamide, probably due to the metabolite,
chloroacetaldehyde.
Secondary malignancies may appear years after therapy.
Answer. a

(183). A woman has consumed several tabs of Amitryptiline. All of the following can be used as a part of her management except
a. Sodium bicarbonate infusion
b. Gastric lavage
c. Use Atropine as an antidote
d. Diazepam for seizure control

Solution. Ans 183: (c) Use Atropine as an antidote
Ref:Read the text below
Sol:
Poisoning of tricyclic antidepressents
Mode:
Self attempted by depressed patients
Clinical features:
Excitement
Delirium and other anticholinergic symptoms as seen in atropine poisoning
Followed by muscle spasm, convulsion and coma
Respiration is depressed
Body temperature falls
Low BP
Tachycardia
ECG changes and ventricular arrhythmia
Treatment:
Primarily supportive
Gastric lavage
Respiratory support
Fluid infusion
Maintainance of BP and body temperature
Correction of acidosis by bicarbonate infusion
Diazepam for control of convulsion
Propranolol/Lignocaine for ventricular arrhythmia
Physostigmine for correction of anticholinergic symptoms
Answer. c

Copyright 2014 Delhi Academy of Medical Sciences, All Rights Reserved. 78/88
(184). Chemical peeling is done by all except?
a. Trichloroacetic acid
b. Phosphoric acid
c. Carboxylic acid
d. Kezoic acid

Solution. Ans 184: (b) Phosphoric acid
Ref:Read the text below
Sol:
Superficial Chemical peel can include the following ingredients:
Glycolic Acid Peel
Mandelic Acid Peel
Salicylic Acid Peel
Azelaic Acid Peel
Retinol Peel
Kojic Acid Peel
Lacitc Acid Peel
Arginine Peel
Pyruvic Acid Peel

Medium depth chemical peel
TCA Trichloroacetic acid Chemical Peel
Deep chemical peel
Phenol Chemical Peel
Also know
Alpha hydroxy acids (AHAs) are naturally occurring organic carboxylic acids such as glycolic acid, a natural constituent of sugar cane
juice and lactic acid, found in sour milk and tomato juice. This is the mildest of the peel formulas and produces light peels for treatment
of fine wrinkles, areas of dryness, uneven pigmentation and acne. Alpha hydroxy acids can also be mixed with a facial wash or cream in
lesser concentrations as part of a daily skin-care regimen to improve the skin's texture.
There are five main fruit acids: citric acids (citrus-derived), glycolic (derived from sugar cane), lactic acid (although derived from milk,
this is still considered a "fruit acid"), malic (derived from apples) and tartaric (derived from grapes).
Answer. b

Copyright 2014 Delhi Academy of Medical Sciences, All Rights Reserved. 79/88
(185). Respiratory centre depression is caused by all except
a. Opium
b. Strychnine
c. Barbiturates
d. Gelsenium

Solution. Ans 185: (b) Strychnine
Ref:Read the text below
Sol:
Opoids and Barbiturates are well known drugs causing respiratory depression
Morphine-like opioids depress respiration at least in part by virtue of a direct effect on the brainstem respiratory centers.
The respiratory depression is discernible even with doses too small to disturb consciousness and increases progressively as the dose is
increased. In humans, death from morphine poisoning is nearly always due to respiratory arrest.
Therapeutic doses of morphine in humans depress all phases of respiratory activity (rate, minute volume, and tidal exchange) and also
may produce irregular and periodic breathing.
Barbiturates depress both the respiratory drive and the mechanisms responsible for the rhythmic character of respiration.
The neurogenic drive is diminished by hypnotic doses but usually no more so than during natural sleep.
However, neurogenic drive is essentially eliminated by a dose three times greater than that used normally to induce sleep. Such doses
also suppress the hypoxic drive and, to a lesser extent, the chemoreceptor drive.
At still higher doses, the powerful hypoxic drive also fails. However, the margin between the lighter planes of surgical anesthesia and
dangerous respiratory depression is sufficient to permit the ultra-short-acting barbiturates to be used, with suitable precautions, as
anesthetic agents.
Gelsenium
In its homeopathic form, Gelsemium is prepared from the fresh root, which is chopped, soaked in alcohol, strained, and diluted to the
desired, highly dilute potencies.
Homeopathic physicians prescribe dilute solutions of gelsemium for ailments that are accompanied by symptoms like those of
gelsemium poisoning.
Gelsemium mainly affects the brain and spinal cord, the motor nerves, muscles, eyelids, and mucous membranes.
Gelsemium alleviates the following: headaches that intensify with movement or bright light and feel as if there is a tight band around
the head; a sore scalp due to nerve inflammation; muscle pain associated with fever nervous disorders, such as multiple sclerosis; uterine
pain and painful menstruation; achy right-eye pain; a sore throat with red tonsils, difficulty swallowing and earache; and summer colds.
It is given for feverish symptoms, such as a sweaty, flushed face, an unpleasant taste in the mouth, a furred, trembling tongue,
twitching muscles that feel cold and tingly, chills with waves of heat along the spine.
Gelsemium is also used for fears, such as surgery or going to the dentist, and fear after a shock. All are accompanied by trembling.
Overexcitement that causes the heart to miss a beat, drowsiness, and insomnia are also helped by Gelsemium.
Excessive doses cause respiratory depression.
Strychnine
Ten to twentyminutes after exposure, the body's muscles begin to spasm, starting with the head and neck in the form of trismus and
risus sardonicus.
The spasms then spread to every muscle in the body, with nearly continuous convulsions, and get worse at the slightest stimulus.
The convulsions progress, increasing in intensity and frequency until the backbone arches continually.
Convulsions lead to lactic acidosis, hyperthermia and rhabdomyolysis. These are followed by postictal depression .
Death comes from asphyxiation caused by paralysis of the neural pathways that control breathing, or by exhaustion from the
convulsions.
The subject dies within 23 hours after exposure.
Answer. b

Copyright 2014 Delhi Academy of Medical Sciences, All Rights Reserved. 80/88
(186). All cause SLE like syndrome except?
a. INH
b. Penicillin
c. Hydralazine
d. Sulphonamide

Solution. Ans 186: (b) Penicillin
Ref:Read the text below
Sol:
Drug-Induced Lupus
This is a syndrome of positive ANA associated with symptoms such as fever, malaise, arthritis or intense arthralgias/myalgias,
serositis, and/or rash.
The syndrome appears during therapy with certain medications and biologic agents has less female predilection than SLE, rarely
involves kidneys or brain, is rarely associated with anti-dsDNA, is commonly associated with antibodies to histones, and usually
resolves over several weeks after discontinuation of the offending medication.
Among the most frequent are
a. The antiarrhythmics procainamide Disopyramide, and propafenone
b.The antihypertensive hydralazine
c. Several angiotensin-converting enzyme inhibitors and beta blockers
d.The antithyroid propylthiouracil
e.the antipsychotics chlorpromazine and lithium
f. The anticonvulsants carbamazepine and phenytoin
g. The antibiotics isoniazid, minocycline, and macrodantin
h.The antirheumatic sulfasalazine
i. The diuretic hydrochlorothiazide
j. The antihyperlipidemics lovastatin and simvastatin
k. Interferons and tnf inhibitors.
Answer. b

(187). In over active bladder treatment from choice is
a. Darifenacine
b. Duloxetine
c. Oxybutinin hydrochloride
d. Flavoxate

Solution. Ans 187: (a) Darifenacine
Ref:Read the text below
Sol:
Both Darifenacin and Oxybutynin are used but darifenacin more selective .
Oxybutynin, Tolterodine, trospium chloride propiverinehydrochloride, solifenacin , darifenacin and Ditropan patch are
anticholinergic agents to be used to treat detrusor overactivity;
No head-to-head trials ofthese agents have assessed efficacy and side effects. Based on most of the available literature, these agents
are clinically similar, and none appears to offer a major distinct advantage over the others.However, slight differences in these agents
may be clinically useful in drug selection.
The effects of oxybutynin are not tissue-specific .
Darifenacin has the most selective M
3
activity and has shown the greatest degree of safety regarding qTc interval changes and cognitive
impairment, suggesting it may offer a slight advantage in elderly patients and in those with early dementia or cardiac arrhythmias.
Tolterodine has a greater inhibitory effect on bladder contraction than on salivation. Therefore, it has fewer side effects (eg, dryness of
mouth), but with comparable efficacy.
The main advantage of solifenacin is a long half-life, which may be helpful in some patients.
Answer. a

Copyright 2014 Delhi Academy of Medical Sciences, All Rights Reserved. 81/88
(188). Methotrexate resistance occurs by
a. Overactivity of the DHFR
b. Decrease folate reductase
c. Mutation on thymine kinase
d. Defect thymidilate kinase

Solution. Ans 188: (a) Overactivity of the DHFR
Ref:Read the text below
Sol:
Nonproliferating cells are resistant to MTX, probably because of a relative lack of DHFR, thymidylate synthase, and/or the
glutamylating enzyme.
Decreased levels of the MTX polyglutamate have been reported in resistant cells and may be due to its decreased formation or
increased breakdown.
Resistance in neoplastic cells can be due to amplification (production of additional copies) of the gene that codes for DHFR, resulting
in increased levels of this enzyme.
The enzyme affinity for MTX may also be diminished. Resistance can also occur from a reduced influx of MTX, apparently caused by a
change in the carrier-mediated transport
Answer. a

Copyright 2014 Delhi Academy of Medical Sciences, All Rights Reserved. 82/88
(189). Child With phacomelia anomaly due to drug taken by mother
a. Tetracycline
b. Thalidomide
c. Chloroquin
d. Quinolones

Solution. Ans 189: (b) Thalidomide
Ref:Read the text below
Sol:
Agents Acting on Pregnant Women That May Adversely Affect the Structure or Function of the Fetus and Newborn
DRUG EFFECT ON FETUS
Accutane (isotretinoin) Facial-ear anomalies, heart disease, CNS anomalies
Alcohol Congenital cardiac, CNS, limb anomalies; IUGR; developmental delay; attention deficits; autism
Aminopterin Abortion, malformations
Amphetamines Congenital heart disease, IUGR, withdrawal
Azathioprine Abortion
Busulfan Stunted growth; corneal opacities; cleft palate; hypoplasia of ovaries, thyroid, and parathyroids
Carbamazepine Spina bifida, possible neurodevelopmental delay
Carbimazole Scalp defects, choanal atresia, esophageal atresia, developmental delay
Carbon monoxide Cerebral atrophy, microcephaly, seizures
Chloroquine Deafness
Chorionic villus sampling Probably no effect, possibly limb reduction
Cigarette smoking Low birthweight for gestational age
Cocaine/crack Microcephaly, LBW, IUGR, behavioral disturbances
Cyclophosphamide Multiple malformations
Danazol Virilization
17-Ethinyl testosterone Masculinizaition of female fetus
Hyperthermia Spina bifida
Lithium Ebstein anomaly, macrosomia
6-Mercaptopurine Abortion
Methyl mercury Minamata disease, microcephaly, deafness, blindness, mental retardation
Methyltestoserone Masculinization of female fetus
Misoprostol Arthrogryposis, cranial neuropathies (Mbius syndrome), equinovarus
Norethindrone Masculinization of female fetus
Penicillamine Cutis laxa syndrome
Phenytoin Congenital anomalies, IUGR, neuroblastoma, bleeding (vitamin K deficiency)
Polychlorinated biphenyls Skin discolorationthickening, desquamation, LBW, acne, developmental delay
Prednisone Oral clefts
Progesterone Masculinization of female fetus
Quinine Abortion, thrombocytopenia, deafness
Statins IUGR, limb deficiencies, VACTERAL
Stilbestrol Vaginal adenocarcinoma in adolescence
Streptomycin Deafness
Tetracycline
Retarded skeletal growth, pigmentation of teeth, hypoplasia of enamel, cataract, limb
malformations
Thalidomide Phocomelia, deafness, other malformations
Toluene Craniofacial abnormalities, prematurity, withdrawal symptoms, hypertonia
Trimethadione and paramethadione Abortion, multiple malformations, mental retardation
Valproate CNS, facial and cardiac anomalies, limb defects, impaired neurologic function
Vitamin D Supravalvular aortic stenosis, hypercalcemia
Warfarin Fetal bleeding and death, hypoplastic nasal structures
Answer. b
Copyright 2014 Delhi Academy of Medical Sciences, All Rights Reserved. 83/88

(190). About Ramelteon all are true except
a. Both MT1, MT2 agonist
b. Addiction liability
c. FDA approved for t/t of insomnia
d. Cyp50 inhibitor

Solution. Ans 190: (b) Addiction liability
Ref:Read the text below
Sol:
Ramelteon
Melatonin receptors are thought to be involved in maintaining circadian rhythms underlying the sleep-wake cycle .
Ramelteon, a novel hypnotic drug prescribed specifically for patients who have difficulty in falling asleep, is an agonist at MT
1
and
MT
2
melatonin receptors located in the suprachiasmatic nuclei of the brain.
The drug has no direct effects on GABAergic neurotransmission in the central nervous system.
In polysomnography studies of patients with chronic insomnia, ramelteon reduced the latency of persistent sleep with no effects on
sleep architecture and no rebound insomnia or significant withdrawal symptoms.
The drug is rapidly absorbed after oral administration and undergoes extensive first-pass metabolism, forming an active metabolite
with longer half-life (25 hours) than the parent drug.
The CYP1A2 isoform of cytochrome P450 is mainly responsible for the metabolism of ramelteon, but the CYP2C9 isoform is also
involved.
The drug should not be used in combination with inhibitors of CYP1A2 (eg, ciprofloxacin, fluvoxamine, tacrine, zileuton) or CYP2C9
(eg, fluconazole) and should be used with caution in patients with liver dysfunction.
The CYP inducer rifampin markedly reduces the plasma levels of both ramelteon and its active metabolite.
Adverse effects of ramelteon include dizziness, somnolence, fatigue, and endocrine changes as well as decreases in testosterone and
increases in prolactin.
Ramelteon is not a controlled substance
Answer. b

Copyright 2014 Delhi Academy of Medical Sciences, All Rights Reserved. 84/88
(191). Woman treated with Lithium during pregnancy should be tested for
a. Neural tube defects
b. Cardiac anomalies
c. Urogenital anomalies
d. Facial deformities

Solution. Ans 191: (b) Cardiac anomalies
Ref:Read the text below
Sol:
Agents Acting on Pregnant Women That May Adversely Affect the Structure or Function of the Fetus and Newborn
DRUG EFFECT ON FETUS
Accutane (isotretinoin) Facial-ear anomalies, heart disease, CNS anomalies
Alcohol Congenital cardiac, CNS, limb anomalies; IUGR; developmental delay; attention deficits; autism
Aminopterin Abortion, malformations
Amphetamines Congenital heart disease, IUGR, withdrawal
Azathioprine Abortion
Busulfan Stunted growth; corneal opacities; cleft palate; hypoplasia of ovaries, thyroid, and parathyroids
Carbamazepine Spina bifida, possible neurodevelopmental delay
Carbimazole Scalp defects, choanal atresia, esophageal atresia, developmental delay
Carbon monoxide Cerebral atrophy, microcephaly, seizures
Chloroquine Deafness
Chorionic villus sampling Probably no effect, possibly limb reduction
Cigarette smoking Low birthweight for gestational age
Cocaine/crack Microcephaly, LBW, IUGR, behavioral disturbances
Cyclophosphamide Multiple malformations
Danazol Virilization
17-Ethinyl testosterone Masculinizaition of female fetus
Hyperthermia Spina bifida
Lithium Ebstein anomaly, macrosomia
6-Mercaptopurine Abortion
Methyl mercury Minamata disease, microcephaly, deafness, blindness, mental retardation
Methyltestoserone Masculinization of female fetus
Misoprostol Arthrogryposis, cranial neuropathies (Mbius syndrome), equinovarus
Norethindrone Masculinization of female fetus
Penicillamine Cutis laxa syndrome
Phenytoin Congenital anomalies, IUGR, neuroblastoma, bleeding (vitamin K deficiency)
Polychlorinated biphenyls Skin discolorationthickening, desquamation, LBW, acne, developmental delay
Prednisone Oral clefts
Progesterone Masculinization of female fetus
Quinine Abortion, thrombocytopenia, deafness
Statins IUGR, limb deficiencies, VACTERAL
Stilbestrol Vaginal adenocarcinoma in adolescence
Streptomycin Deafness
Tetracycline
Retarded skeletal growth, pigmentation of teeth, hypoplasia of enamel, cataract, limb
malformations
Thalidomide Phocomelia, deafness, other malformations
Toluene Craniofacial abnormalities, prematurity, withdrawal symptoms, hypertonia
Trimethadione and paramethadione Abortion, multiple malformations, mental retardation
Valproate CNS, facial and cardiac anomalies, limb defects, impaired neurologic function
Vitamin D Supravalvular aortic stenosis, hypercalcemia
Warfarin Fetal bleeding and death, hypoplastic nasal structures
Answer. b
Copyright 2014 Delhi Academy of Medical Sciences, All Rights Reserved. 85/88

(192). Oryxenergic neuron secrete nueropeptide Y . which of the following is the true statement
a. Its secretion controlled by melotonic receptor
b. Constitutes 32 aminoacid
c. Increases in starvation
d. Decreases thermogenesis

Solution. Ans 192: (c) Increases in starvation
Ref:Read the text below
Sol:
Neuropeptide Y is a polypeptide containing36 amino acid residues that acts on at least two of the four known G protein-coupled
receptors: Y1, Y2, Y4, and Y5.
Neuropeptide Y is found throughout the brainand the autonomic nervous system.
When injected into the hypothalamus, this polypeptide increases food intake, and inhibitors of neuropeptide Y synthesis decrease
food intake.
Neuropeptide Y-containing neurons have their cellbodies in the arcuate nuclei and project to the paraventricular nuclei
Answer. c

(193). A patient has completed a course of cancer chemotherapy and now has severe anemia, neutropenia, and thrombocytopenia. If only one
intervention is possible, which of the following is the most appropriate therapy?
a. Epoetin
b. Filgrastim
c. Growth hormone
d. Sargramostim

Solution. Ans-193: (d) Sargramostim
Ref:Read the text below
Sol:
Apatient who is anemic, neutropenic, and thrombocytopenic requires stimulation of all three major cell lines in the bone marrow.
The only drug currently available that accomplishes this broad-spectrum stimulant effect is sargramostim
(granulocytemacrophage colony stimulating factor [GM-CSF]).
Epoetin is a more selective stimulant of erythrocyte production and is useful in simple anemia.
Filgrastim is a somewhat selective stimulant of leukocyte production and has much less effect on erythrocytes and platelet
production than sargramostim
Growth hormone and testosterone have both been tried in the treatment of anemia with negligible success.
Ans-193: (d) Sargramostim
Ref:Read the text below
Sol:
Apatient who is anemic, neutropenic, and thrombocytopenic requires stimulation of all three major cell lines in the bone marrow.
The only drug currently available that accomplishes this broad-spectrum stimulant effect is sargramostim
(granulocytemacrophage colony stimulating factor [GM-CSF]).
Epoetin is a more selective stimulant of erythrocyte production and is useful in simple anemia.
Filgrastim is a somewhat selective stimulant of leukocyte production and has much less effect on erythrocytes and platelet
production than sargramostim
Growth hormone and testosterone have both been tried in the treatment of anemia with negligible success.
Answer. d

Copyright 2014 Delhi Academy of Medical Sciences, All Rights Reserved. 86/88
(194). Which of the following is the agent of choice for chronic treatment of simple hypothyroidism (myxedema)?
a. Desiccated thyroid
b. Levothyroxine (T4)
c. Liothyronine (T3)
d. Potassium iodide

Solution. Ans-194: (b) Levothyroxine (T4)
Ref:Read the text below
Sol:
The agent of choice in treating simple hypothyroidism is levothyroxine (T4).
Administered thyroxine is bound in plasma in the same way and metabolized to tri-iodothyronine in the same manner as
native thyroxine.
Although liothyronine (tri-iodothyronine [T3]) ismore active than T4 as an agonist at thyroid hormone receptors, its shorter half-life of
1 day as compared with the 67-day half-life of T4 means that thyroxine has a longer duration of action and provides smoother
control.
Answer. b

(195). Which of the following correctly describes the mechanism of action of streptokinase?
a. Combines with plasminogen to form an enzymatically active complex
b. Competitively Blocks Binding Of Plasminogen To Fibrin
c. Converts plasmin to plasminogen
d. Inhibits platelet cyclooxygenase (COX) activity

Solution. Ans-195: (a) Combines with plasminogen to form an enzymatically active complex
Ref:Read the text below
Sol:
Streptokinase has no intrinsic enzymatic activity, but instead forms a stable complex with the patients plasminogen, making it
enzymatically active in cleaving free plasminogen to plasmin.
The streptokinase plasminogen complex is not inhibited by antiplasmin. The other thrombolytic agentst-PA, reteplase,
tenecteplase, and urokinase activate plasminogen directly.
Answer. a

(196). A17-year-old patient suffers from tonic-clonic seizures. This condition has been well controlled with a regimen of phenytoin. Which of the
following signs or symptoms indicates phenytoin toxicity?
a. Diplopia and abnormal gait
b. Hyperprolactinemia
c. Polydipsia and polyuria
d. Postural hypotension
e. Rigidity and tremor

Solution. Ans-196: (a) Diplopia and abnormal gait
Ref:Read the text below
Sol:
Diplopia, abnormal gait, and other signs of cerebellar dysfunction are important symptoms of phenytoin toxicity.
Other manifestations of toxicity include gingival hyperplasia, nystagmus,and vertigo. Hyperprolactinemia is an adverse effect of
antipsychotic dopamine antagonists such as the phenothiazines; dopamine inhibits prolactin secretion by the anterior
pituitary. Polydipsia and polyuria are symptoms of diabetes insipidus.
These symptoms may be produced by lithium toxicity during treatment of bipolar depression, and are not associated with phenytoin
toxicity.
Postural hypotension is not an adverse effect of phenytoin but often occurs with levodopa treatment of Parkinsons disease.
Answer. a

Copyright 2014 Delhi Academy of Medical Sciences, All Rights Reserved. 87/88
(197). Which of the following substances contains the highest concentration of molecules with estrogenic effects?
a. Ginseng
b. Grapefruit juice
c. Saw palmetto
d. Tomatoes

Solution. Ans-197: (c) Saw palmetto
Ref:Read the text below
Sol:
Saw palmetto is promoted by purveyors of herbs and alternative medicines as therapy for prostatic hyperplasia and does have weak
estrogenic activity.
Grapefruit juice and tomatoes are used therapeutically in a treatment regimen referred to as raw juice therapy.
Answer. c

(198). Calcium- and aluminum-containing antacids should be avoided by patients taking which of the following antimicrobial drugs?
a. Amoxicillin
b. Erythromycin
c. Isoniazid
d. Minocycline

Solution. Ans-198: (d) Minocycline
Ref:Read the text below
Sol:
Tetracyclines such as minocyclinechelatecations, such as aluminum and calcium, andmay precipitate in the gut, greatly
reducingantibiotic bioavailability.
Answer. d

(199). A young boy was treated for 2 years with several antibacterial agents to eradicate tuberculosis.Upon investigation, he is found to have
profound hearing loss. Which of the following agents may have been responsible for this hearing loss?
a. Ethambutol
b. Isoniazid
c. Pyrazinamide
d. Streptomycin

Solution. Ans-199: (d) Streptomycin
Ref:Read the text below
Sol:
The agents listed are the primary drugs used in the treatment of tuberculosis. Streptomycin and other aminoglycosides cause
eighth nerve damage, which is often irreversible and may take the form of auditory or vestibular dysfunction.
Isoniazid and rifampinare the most efficacious and least toxic, but even when used together are insufficient in many cases to
prevent the development of resistance.
Therefore, most cases of tuberculosis are treatedwith three or even four agents in an effort to eradicate the infection before
resistance develops.
Answer. d

Copyright 2014 Delhi Academy of Medical Sciences, All Rights Reserved. 88/88
(200). Which of the following drugs exerts its effects through inhibition of cyclic GMP phosphodiesterase?
a. Hydralazine
b. Minoxidil
c. Nitroprusside
d. Sildenafil

Solution. Ans-200: (d) Sildenafil
Ref:Read the text below
Sol:
Vasodilators act by one of three mechanisms: increasing cyclic GMP (cGMP) levels in vascular smooth muscle cells; opening
potassium channels; or blocking calcium channels.
The organic nitrates and nitroprusside (nitrovasodilators) increase cGMP synthesis by generating nitric oxide (NO), which
subsequently activates a soluble form of guanylyl cyclase.
Activation of muscarinic receptors on vascular endothelial cells results in formation of NO (earlier identified as endothelium-
derived relaxing factor) that diffuses to smooth muscle cells and relaxes them through increased cGMP levels.
Erection of the penis involves neuronally regulated formation of NO, increased cGMP levels in the corpus cavernosum, and
relaxation of cavernosal and vascular smooth muscle in erectile tissue.
Rather than stimulating guanylyl cyclase, sildenafil (Viagra) acts as a selective inhibitor of cGMP phosphodiesterase type 5 to increase
the half-life of cGMP in the tissues.
Answer. d
Test Answer

1.(a) 2.(b) 3.(c) 4.(a) 5.(a) 6.(a) 7.(b) 8.(d) 9.(a) 10.(a)
11.(c) 12.(b) 13.(d) 14.(b) 15.(c) 16.(d) 17.(d) 18.(d) 19.(c) 20.(d)
21.(b) 22.(b) 23.(a) 24.(b) 25.(a) 26.(a) 27.(a) 28.(c) 29.(a) 30.(d)
31.(d) 32.(b) 33.(c) 34.(b) 35.(b) 36.(d) 37.(b) 38.(b) 39.(c) 40.(d)
41.(d) 42.(c) 43.(c) 44.(c) 45.(b) 46.(c) 47.(a) 48.(b) 49.(c) 50.(d)
51.(c) 52.(c) 53.(c) 54.(d) 55.(c) 56.(b) 57.(b) 58.(b) 59.(d) 60.(d)
61.(b) 62.(b) 63.(d) 64.(c) 65.(d) 66.(b) 67.(b) 68.(c) 69.(d) 70.(a)
71.(d) 72.(a) 73.(b) 74.(c) 75.(d) 76.(c) 77.(c) 78.(d) 79.(a) 80.(c)
81.(c) 82.(a) 83.(b) 84.(a) 85.(d) 86.(b) 87.(d) 88.(d) 89.(d) 90.(c)
91.(b) 92.(b) 93.(a) 94.(d) 95.(c) 96.(c) 97.(c) 98.(b) 99.(c) 100.(a)
101.(b) 102.(b) 103.(b) 104.(a) 105.(a) 106.(b) 107.(b) 108.(d) 109.(b) 110.(c)
111.(a) 112.(a) 113.(b) 114.(d) 115.(c) 116.(d) 117.(b) 118.(d) 119.(b) 120.(d)
121.(c) 122.(c) 123.(d) 124.(b) 125.(c) 126.(c) 127.(a) 128.(b) 129.(d) 130.(d)
131.(d) 132.(d) 133.(a) 134.(c) 135.(b) 136.(d) 137.(d) 138.(d) 139.(d) 140.(b)
141.(a) 142.(d) 143.(d) 144.(d) 145.(d) 146.(b) 147.(b) 148.(c) 149.(d) 150.(c)
151.(c) 152.(d) 153.(b) 154.(b) 155.(c) 156.(c) 157.(a) 158.(b) 159.(a) 160.(d)
161.(b) 162.(d) 163.(b) 164.(b) 165.(c) 166.(b) 167.(b) 168.(a) 169.(d) 170.(c)
171.(a) 172.(b) 173.(d) 174.(a) 175.(a) 176.(b) 177.(d) 178.(b) 179.(b) 180.(c)
181.(d) 182.(a) 183.(c) 184.(b) 185.(b) 186.(b) 187.(a) 188.(a) 189.(b) 190.(b)
191.(b) 192.(c) 193.(d) 194.(b) 195.(a) 196.(a) 197.(c) 198.(d) 199.(d) 200.(d)

You might also like